OJ EPO SE 5/2014, p25 - PART II

BOARD OF APPEAL AND ENLARGED BOARD OF APPEAL CASE LAW 2012/2013

I. PATENTABILITY

A. Exceptions to patentability

1. Breaches of "ordre public" or morality

Under Art. 53(a) EPC, European patents are not granted in respect of inventions the commercial exploitation of which would be contrary to "ordre public" or morality.

In T 2221/10, the board had to examine whether the subject-matter of the appellant's (applicant's) request fell under the exceptions to patentability of Art. 53(a) EPC in conjunction with R. 28(c) EPC. Claims 1 and 2 of appellant's sole request referred to methods of maintaining human embryonic stem cells (HES cells) in culture in an undifferentiated state. Claim 5 referred to a cell culture comprising HES cells. HES cells are derived from the inner cell mass of human embryos at the blastocyst stage and can be proliferated in vitro in an undifferentiated state. They are capable of developing into any organ or tissue of the human body.

The appellant argued that methods using commercially or otherwise publicly available HES cell lines were not excluded from patentability because no de novo destruction of human embryos was necessary to perform them.

Regarding the appellant's argument that it would go too far if one were to take into account all the steps preceding an invention when assessing exclusion from patentability, the board referred to G 2/06, OJ EPO 2009, 306, point 23 of the Reasons:

"In a case like the present one, where the teaching to obtain the embryonic human stem cells claimed is confined to the use (involving their destruction) of human embryos, the argument raised by the Appellant, namely that the exclusion from patentability would go much too far if one would consider all the steps preceding an invention for the purposes of R. 28(c) EPC (formerly R. 23d(c) EPC) is not relevant."

The board interpreted this statement as meaning that for the purpose of R. 28(c) EPC, all steps preceding the claimed use of HES cells which are a necessary precondition for carrying out the claimed invention, have to be considered. In this respect the Enlarged Board of Appeal had neither made a distinction between steps which have been carried out by the inventor or by any other person, nor between steps which took place in direct preparation of the experiments leading to an invention and steps having taken place at a point in time further remote from these experiments.

The board decided that inventions, which make use of HES cells obtained by de novo destruction of human embryos or of publicly available HES cell lines which were initially derived by a process resulting in the destruction of the human embryos, are excluded from patentability under the provisions of Art. 53(a) EPC in combination with R. 28(c) EPC. It noted that its decision was in line with decision C-34/10 of the ECJ.

In T 1836/10, claim 1 was directed to a method for isolating pluripotent embryonic stem cells without destroying the embryo. The base material used in the method was blastocysts, i.e. embryos about 4 to 7 days old and already showing differentiation, which could be taken e.g. from a domesticated animal or a primate such as a human being. Claim 1 included a disclaimer excluding the stem cells' industrial or commercial use if the blastocyst used was human.

The board found that the use of the product obtained by the method i.e. the embryonic stem cells was not relevant for determining the claimed subject-matter, because their disclosed possible future uses were to be regarded not as an additional method step or as a further technical feature of the claimed method, but merely as "subjective intentions". Therefore, the disclaimer did not limit the subject-matter in any way. The board ruled that claim 1 comprised subject-matter which infringed Art. 53(a) EPC in conjunction with R. 28(c) EPC, because the use of human embryos as base material in an industrially applied method for isolating embryonic stem cells was to be regarded as "use for industrial or commercial purposes" within the meaning of R. 28(c) EPC (see also decision C-34/10 of the ECJ concerning the interpretation of Art. 6(2)(c) of Directive 98/44/EC of 6 July 1998 on the legal protection of biotechnological inventions, especially point 3 of the judgment's operative part).

2. Patentability of biological inventions

2.1 Essentially biological processes for the production of plants

Under Art. 53(b) EPC, patents are not granted in respect of plant varieties or essentially biological processes for the production of plants. This exclusion does not apply to microbiological processes or their products.

In its second interlocutory decision in T 1242/06 of 31 May 2012 (OJ EPO 2013, 42), the board expressed serious concern that allowing claims to plant material obtained by means of an essentially biological breeding process would de facto frustrate the legislator's intentions in framing the process exclusion, which had been highlighted in decision G 1/08 (OJ EPO 2012, 206), and in many cases make circumvention of the exclusion a matter of skilful claim drafting, thereby diminishing the consistency and persuasiveness of the legislative framework of the EPC as regards patentable subject-matter. The board was of the opinion that these issues raised points of law of fundamental importance and referred the following questions to the Enlarged Board of Appeal for decision:

(1) Can the exclusion of essentially biological processes for the production of plants in Art. 53(b) EPC have a negative effect on the allowability of a product claim directed to plants or plant material such as a fruit?

(2) In particular, is a claim directed to plants or plant material other than a plant variety allowable even if the only method available at the filing date for generating the claimed subject-matter is an essentially biological process for the production of plants disclosed in the patent application?

(3) Is it of relevance in the context of questions 1 and 2 that the protection conferred by the product claim encompasses the generation of the claimed product by means of an essentially biological process for the production of plants excluded as such under Art. 53(b) EPC?

The case is pending as G 2/12.

In T 83/05 of 8 July 2013, the subject-matter of claim 1 of the respondent's main request was defined as an edible Brassica plant produced according to a certain method for the production of Brassica oleracea with elevated levels of certain glucosinolates.

For the same reasons as set out in great detail in the second interlocutory decision in case T 1242/06, the board considered that the plants or parts of plants claimed did not fall under the exclusion of plant varieties as stipulated in Art. 53(b) EPC and defined by R. 26(4) EPC.

There was nevertheless, similarly to case T 1242/06, a second issue which needed to be addressed, namely the question as to whether the process exclusion contained in Art. 53(b) EPC had a negative impact on the allowability of the respondent's product claims. In the board's view, at least the method steps referred to in claims 1 to 3 of the main request for the definition of the claimed plants and plant parts had to be regarded as essentially biological processes for the production of plants and would fall, if claimed as such, under the process exclusion of Art. 53(b) EPC. These claims therefore covered plants and plants parts which were produced by an excluded process.

In view of the principle of absolute product protection (see G 2/88), a claim to a product provides the patent proprietor with protection that generally encompasses the protection provided by a patent claim for the process of making the product (see decision G 2/06). If the product claims were allowed in the present case, any act of making the claimed Brassica and broccoli plants or plant parts would in principle fall under the prerogative of the patent proprietor.

This would have the consequence that the proprietor could prevent others from carrying out the breeding method taught in the description of the patent and referred to in the claims, although this method might be regarded as an essentially biological process for the production of plants and be excluded per se from patentability under Art. 53(b) EPC.

The board referred the following questions to the Enlarged Board of Appeal:

1. Can the exclusion of essentially biological processes for the production of plants in Art. 53(b) EPC have a negative effect on the allowability of a product claim directed to plants or plant material such as plant parts?

2. In particular:

(a) Is a product-by-process claim directed to plants or plant material other than a plant variety allowable if its process features define an essentially biological process for the production of plants?

(b) Is a claim directed to plants or plant material other than a plant variety allowable even if the only method available at the filing date for generating the claimed subject-matter is an essentially biological process for the production of plants disclosed in the patent application?

3. Is it of relevance in the context of questions 1 and 2 that the protection conferred by the product claim encompasses the generation of the claimed product by means of an essentially biological process for the production of plants excluded as such under Art. 53(b) EPC?

4. If a claim directed to plants or plant material other than a plant variety is considered not allowable because the plant product claim encompasses the generation of the claimed product by means of a process excluded from patentability under Art. 53(b) EPC, is it possible to waive the protection for such generation by "disclaiming" the excluded process?

The case is pending as G 2/13 and has been consolidated with G 2/12.

3. Medical methods

Under Art. 53(c) EPC, patents are not granted in respect of methods for treatment of the human or animal body by surgery or therapy and diagnostic methods practised on the human or animal body.

3.1 Surgical methods

In T 1798/08, although all claims were directed to a visual prosthesis, i.e. an apparatus, they were objected to by the opponent under Art. 53(c) EPC as relating to a surgical method. The features "suitable to be located on the body of the user outside a wall of the sclera and attached to the sclera", and "suitable to be located implanted in the eye behind the iris" were regarded as relating to a method for treatment of the human or animal body by surgery, thus transforming the claim into a "disguised" method claim, even though it was notionally directed to a device.

The board dismissed this objection. It held that Art. 53(c), second sentence, EPC specifies that the provision does not apply to products, e.g. substances and compositions, for use in the methods falling under the exception clause. In addition to substances and compositions, the claim category "products" includes apparatus. Accordingly, the provisions of Art. 53(c) EPC do not normally apply to apparatus claims. The fact that some features of the claimed apparatus are functionally defined in relation to the body of the patient does not itself transform the apparatus claim into a method claim (T 712/93, T 1695/07). It is true that the actual implantation of components of the prosthesis would constitute a surgical intervention in the patient's body, but this was not what was claimed in claim 1. The claim merely defined that certain components of the prosthesis are "suitable to be located" at various locations in the patient's body. Such a definition does not exclude the claimed apparatus from patentability under Art. 53(c) EPC.

The board distinguished the case from T 775/97 and T 82/93 (OJ EPO 1996, 274).

B. Novelty

1. Defining the state of the art

1.1 Availability to the public

Information is said to be "available" to the public if only a single member of the public is in a position to gain access to it and understand it, and if there is no obligation to maintain secrecy.

In T 834/09 the question to be decided was whether or not document D1 had been made available to the public before the earliest priority date of the contested patent, i.e. before 23 April 1996. Document D1 was received and date-stamped by the University of California Libraries San Diego on 3 April 1996. However, there was no evidence whatsoever that D1 was shelved before the relevant date. The board stated that, from the case law of the boards of appeal, it followed by analogy that the person in charge of the reception and date-stamping of an incoming document at a public library is without any doubt a member of the public as this staff member is in no way bound by any obligation to maintain secrecy about the publications he/she handles and the content thereof, and after all, his/her very function as a staff member of a public library is to make information available to the public. The board went on to state that in the case of a written disclosure it is irrelevant whether the staff member is a person skilled in the art or not, because the content of a written disclosure can be freely reproduced and distributed even without understanding it. Thus the board held that the reception and date-stamping of an incoming document by a staff member of a public library makes the document available to the public.

1. 2 Internet disclosures

In T 1553/06 the board developed a test for assessing the public availability of a document stored on the World Wide Web which could be found via a public web search engine on the basis of keywords.

In devising this test the board started from its finding that the mere theoretical possibility of having access to a means of disclosure did not make it become available to the public within the meaning of Art. 54(2) EPC 1973. What is required, rather, is a practical possibility of having access, i.e. "direct and unambiguous access" to the means of disclosure for at least one member of the public as set out in G 1/92 (OJ EPO 1993, 277) and T 952/92 (OJ EPO 1995, 755). In the case of a document stored on the World Wide Web which can only be accessed by guessing a Uniform Resource Locator (URL) not made available to the public, "direct and unambiguous access" to the document is possible in exceptional cases only, i.e. where the URL is so straightforward, or so predictable, that it can readily be guessed. The fact that a document stored on the World Wide Web could be found by entering keywords in a public web search engine before the priority or filing date of the patent or patent application is not always sufficient to reach the conclusion that "direct and unambiguous access" to the document was possible. The test is as follows: where all the conditions set out in the following test are met, it can be safely concluded that a document stored on the World Wide Web has been made available to the public:

If, before the filing or priority date of the patent or patent application, a document stored on the World Wide Web and accessible via a specific URL

(1) could be found with the help of a public web search engine by using one or more keywords, all related to the essence of the content of that document, and

(2) remained accessible at that URL for a period of time long enough for a member of the public, i.e. someone under no obligation to keep the content of the document secret, to have direct and unambiguous access to the document, then the document has been made available to the public within the meaning of Art. 54(2) EPC 1973.

However, it should be noted that if either of conditions (1) and (2) is not met, the above test does not permit any conclusion to be drawn on whether or not the document in question has been made available to the public.

In T 2/09 the board stated that it had doubts as to whether public availability of emails transmitted via the internet could reasonably be established at all if the technical conditions of the test in T 1553/06 for public availability of web pages were to be applied mutatis mutandis, i.e. whether emails transmitted over the internet could be accessed and searched in a way comparable to that of web pages, independent of whether or not access to and disclosure of the content of the email were lawful. The board was rather of the opinion that the differences between web pages and such emails make a strong prima facie case against public availability of the latter. The board decided that the content of an email did not become available to the public within the meaning of Art. 54(2) EPC 1973 for the sole reason that the email was transmitted via the internet before the filing date of 1 February 2000.

In T 1469/10 the patent proprietor contended that the documents D1, D3 and D14 which were 3GPP meeting contributions uploaded by the ETSI 3GPP organisation on its public file server should not be considered as prior art under Art. 54(2) EPC, as the evidence on file ("timestamps" in 3GPP document lists) did not justify the conclusion that the respective publication dates of those documents were ahead of the filing date of the patent. The board pointed out that the ETSI 3GPP organisation, a reputable standardisation body, had clear and reliable rules for publishing of any meeting contributions, in particular as to the documentation of the uploading to the public file server. Therefore, a publication date indicated on the 3GPP document lists ("timestamp") was of a high probative value and might serve as prima facie evidence as to the date on which the document was available to the public. Accordingly, the board pointed out that the respective dates ("timestamps") indicated on the 3GPP document lists reliably corresponded to the dates on which the respective documents were uploaded to the 3GPP file server. Thus, the documents at issue were regarded as representing the state of the art under Art. 54 (2) EPC.

1.3 Oral disclosure at a lecture

In T 2003/08 the board observed that, in contrast to a written document the contents of which are fixed and can be read again and again, an oral presentation is ephemeral. Therefore, the standard of proof for ascertaining the contents of an oral disclosure is high. What has been said, or to use the terms of Art. 54(2) EPC, what has been "made available to the public" has to be put beyond reasonable doubt. The board noted that in case T 1212/97 the board had expressed the view that "written notes made at the lecture by at least two members of the audience can usually be regarded as sufficient" for that purpose. A fact also indicated by the board was that the amount of evidence necessary to establish the content of an oral presentation beyond reasonable doubt was to be judged on a case-by-case basis, i.e. it depends on the quality of the evidence in each case.

In the current board's view, decision T 1212/97 could not be interpreted as setting an absolute standard for the amount of evidence necessary to prove the contents of an oral disclosure. The board considered that there may be circumstances where evidence from the lecturer and only one member of the audience is convincing enough to reach the standard of proof – i.e. beyond reasonable doubt. In the case at issue, however, evidence from the lecturer and a member of the audience, provided by them in the form of both affidavits and oral testimony, was not considered by the board to prove beyond reasonable doubt that the subject-matter of the claim was disclosed during the lecture.

1.4 Prior use

In T 1682/09 the appellant's allegation of public prior use related to an assembly of a weighing system.

It was noted that, according to the established case law, the selling of an apparatus is, in the absence of any special circumstance, sufficient to render the apparatus sold available to the public. In the case at issue, the assembly was only leased; it was not owned by the company. The board however found that the mounting of the assembly at the premises of a company, the subsequent conventional start-up, training and maintenance procedures of the assembly at the same premises rendered the features of the assembly available to the company, which constituted at that time a member of the public.

2. Issues of proof

2.1 Cases decided on the absolute conviction criterion

In T 71/09 the respondent alleged an instance of public prior use that destroyed the novelty and inventive step of the claimed subject-matter. As suggested by the factual evidence produced, this prior use consisted of the sale of product W. by the respondent's own firm.

The board stated that, according to the established case law of the boards of appeal, a finding that a use forms part of the state of the art for the purpose of Art. 54(2) EPC should only be made if a strict and careful evaluation of the available evidence establishes that prior use occurred (T 750/94, OJ EPO 1998, 32). As a consequence, before finding that a use forms part of the state of the art, the board must be satisfied that the available evidence establishes with almost absolute conviction, i.e. to a degree of certainty beyond all reasonable doubt, that the prior use did indeed take place before the priority date of the patent in suit (T 97/94, OJ EPO 1998, 467). The board held that, in the case in question, the evidence produced by the respondent was not sufficient to establish beyond all reasonable doubt the composition of product W.

3. Determining the content of the relevant prior art

3.1 Taking non-technical features into account when assessing novelty

In T 2050/07 the decision of the examining division had denied the novelty of claim 1 in view of document D6 (D6 was comprised in the state of the art pursuant to Art. 54(3) EPC). The board noted that the argument could be made that the distinguishing features were of a non-technical nature as being a mathematical method or a method for performing mental activities, and that, in view of the established case law according to which features that do not contribute to the technical character of an invention and do not interact with the technical subject-matter of the claim for solving a technical problem have to be ignored when assessing inventive step, such features should equally be ignored when assessing novelty. The board therefore examined whether or not the distinguishing features in the case before it made a technical contribution.

The board considered that the description of the patent application made it sufficiently clear how the distinguishing features (i) and (ii) of the method of claim 1 should be implemented and how they interacted with the remaining steps of the claimed method in order to provide a common technical result. This consideration led the board to consider that the distinguishing features had to be taken into account when assessing the novelty of claim 1 and concluded that the method of claim 1 was new.

4. Novelty of use

4.1 Novelty of the therapeutic application

In T 108/09 the claims were drafted in the second medical use format according to decision G 5/83 (OJ EPO 1985, 64). The board noted that according to decision G 2/08 (OJ EPO 2010, 456), Art. 54(5) EPC does not exclude a medicament already known to be used in the treatment of an illness from being patented for use in a different treatment by therapy of the same illness. This finding applied mutatis mutandis to the Swiss-type claims of the patent in suit (see G 2/08). It considered it appropriate to evaluate whether the breast cancer of claim 1 as granted was identical to the breast cancer according to document D2. The board noted that the tumours of document D2, being only resistant to tamoxifen, could be distinguished from the tumours of claim 1 as granted, which were additionally resistant to an aromatase inhibitor. This distinction meant that in the case at issue two different diseases or two subsets of a disease (tumour) were concerned. As a consequence, in analogy to the findings in T 893/90, the board established novelty. The subject-matter of claim 1 as granted was therefore novel over D2.

4.2 Second (or further) medical use

4.2.1 Differentiation of a direct and indirect effect on claims

In T 1955/09 claim 1 referred to a therapeutic application in the form of the use of a substance or composition for the manufacture of a medicament for a defined therapeutic application (G 5/83, OJ EPO 1985, 64). The attaining of a new technical effect is considered as a functional technical feature of a claim referring to the new use of a known substance. If that technical feature has not been previously made available to the public, then the claimed invention is novel, even though such technical effect may have inherently taken place in the course of carrying out what has previously been made available to the public (see decisions G 2/88, OJ EPO 1990, 93 and G 6/88, OJ EPO 1990, 114).

In the case at issue, the board needed to decide whether the use claimed represented a further and different therapeutic use from the disclosure in document (D1). It stated that the conclusion could not be drawn that the technical effect relied upon by the claimed invention, i.e. the antibiotic effect, was a mere explanation of how the compounds inhibited or neutralized toxins. Rather, this effect identified a new clinical situation, namely one in which it could be preferable to target the infection itself, not merely the toxins produced by the bacteria or fungi causing the infection. The board decided that this reasoning was based on the differentiation of a direct and indirect effect on claims relating to second or further medical uses of a known substance (see i.e. T 836/01 and T 1642/06). In view of the foregoing, the board was satisfied that the subject-matter of claim 1 at issue fulfilled the requirements of Art. 54(1) and (3) EPC vis-à-vis the disclosure in document D1.

4.2.2 Claims for therapeutic methods

In T 454/08 the board confirmed that according to G 5/83 (OJ EPO 1985, 64), a European patent could be granted with claims directed to the use of a substance or composition for the manufacture of a medicament for a specified new and inventive therapeutic application. G 5/83 permitted the reformulation of claims directed to therapeutic methods within the meaning of Art. 52(4) EPC 1973 as Swiss-type claims. In the case in point the board confirmed, however, that if a claim formulated as a Swiss-type claim was not directed to the use of any therapeutic method or was de facto directed to a non-therapeutic use, the feature defining the use was purely illustrative and could not be used to establish novelty over the prior art. In effect, this specific novelty approach applied only to claims directed to the use of a substance or composition intended for use in a method referred to in Art. 52(4) EPC 1973 (now Art. 53(c) EPC).

Claim 1 of the main request was based on the model proposed by G 5/83, namely the application of a substance to produce a composition for a particular use. However, nothing in the wording of claim 1 implied use in any of the therapeutic methods within the meaning of Art. 53(c) EPC. The specific novelty approach created by G 5/83 thus did not apply to claim 1 of the main request, the subject-matter of which amounted to a process claim. The step of administering the tablet had to be seen as an illustrative feature of the tablet and not as a limiting feature for a specific mode of administration.

4.2.3 Means used in the treatment

In T 2003/08 the appellant (opponent) argued, relying on decisions T 227/91 (OJ EPO 1994, 491), T 775/97 and T 138/02 in which the boards defined the term "medicament", that claim 1 did not fulfil one prerequisite to qualify as a second medical use claim in accordance with decision G 5/83 (OJ EPO 1985, 64), namely that a "medicament" was used in the treatment. Indeed the means used in the treatment were a "column", which was not a "medicament", but a "device". Therefore, despite being drafted in the format of a second medical use claim, claim 1 should not be interpreted as such and therefore the treatment feature should be ignored when assessing novelty.

Accordingly, it was necessary to decide whether or not claim 1 was to be construed as a second medical use claim. The question was answered in the affirmative. In the board's view, it emerged from the whole reasoning of decision G 5/83 (i) that the Enlarged Board intended to allow the special second medical use-form of protection only for those uses in the medical domain which concerned a "substance" or "composition", (ii) that it was the "substance" or "composition" which achieved the medical effect and (iii) that the terms "substance" or "composition" referred at least to products which were chemical entities or compositions of chemical entities.

The board consequently considered that it was decisive to establish whether or not the means used to achieve the medical effect were a "substance" or "composition" in the sense of decision G 5/83, rather than to establish whether or not the means were a "medicament". In the case at issue the medical effect on which the treatment according to claim 1 was based was the removal of immunoglobulin from the plasma of patients suffering from dilated cardiomyopathy. This effect was achieved by the "specific ligand for human immunoglobulin", which was undisputedly a chemical entity. The "column" only served as a carrier for the ligand and was not instrumental in achieving the therapeutic effect. In fact, the ligand could also bind immunoglobulin, if it was not bound to the column, i.e. if it was free in solution. Accordingly, the board held that the means used for the treatment in accordance with claim 1 was to be considered as a "substance" or "composition" within the meaning of decision G 5/83.

C. Inventive step

1. Reformulation of the technical problem

In T 1422/12 claim 1 related to crystalline forms of tigecycline. In view of the state of the art, the appellant submitted that the problem underlying the application in suit was the provision of tigecycline which is more stable with respect to epimerisation. The examining division did not accept the formulation of the technical problem as the provision of tigecycline in more thermodynamically stable form, since there was no indication in the application as filed that this was indeed the problem which the invention attempted to solve. The board referred to the established case law, that the technical problem has to be determined on the basis of objectively established facts, since for the determination of the objective technical problem, only the effect actually achieved vis-à-vis the closest prior art should be taken into account (T 13/84, OJ EPO 1986, 253 and T 39/93, OJ EPO 1997, 134). In this connection, any effects may be taken into account, so long as they concern the same field of use and do not change the character of the invention (T 440/91).

In the application it was indicated that the invention in suit related to crystalline forms of tigecycline, tigecycline being a tetracycline antibiotic already marketed as lyophilised powder or cake for intravenous injection, namely in the amorphous form. The section concerning the background of the invention related to improving the performance characteristics of pharmaceutical products, including tigecycline. The formulation of the technical problem to be solved as the provision of tigecycline which is more stable with respect to epimerisation, said reduction in epimerisation resulting in improved biological activity, thus fell within the framework of the invention as disclosed in the application in suit, namely the performance characteristics of the antibiotic, tigecycline, regardless of whether these characteristics were relevant to handling, storage or formulation and/or to its pharmaceutical properties. That the more specific problem of improved stability with respect to epimerisation was not mentioned in the application as originally filed was irrelevant (T 39/93), since improvement of stability by avoidance of epimerisation, and, as a consequence, improved biological activity was clearly recognisable by the skilled person as a desirable effect for a tetracycline antibiotic.

2. Treatment of technical and non-technical features

2.1 Identifying technical features

2.1.1 Shopping with mobile device

In T 1670/07 the application related to a method and system of shopping using a mobile device to obtain a plurality of purchased goods and/or services from a group of vendors located at a shopping location. The invention was essentially that the shopper enters two or more desired goods/services into the mobile device before going shopping and the device displays a shopping itinerary showing an order (sequence) in which the shopper can visit a group of vendors to obtain them. The itinerary is a function of a user profile, e.g. requiring the shortest distance between vendors, or goods at the cheapest purchase price. In the board's view, the overall effect of the method, namely to produce an ordered list of shops, was not technical. The appellant argued that the alleged non-technical feature of the information regarding the group of vendors "interacts with technical elements, in the form of the server, to produce a technical effect in the selection of vendors and the transmission of processed information regarding that selection to the mobile wireless communications device". However, in the board's view, this was an instance of the well-known argument that could be termed the "technical leakage fallacy", in which the intrinsic technical nature of the implementation leaks back into the intrinsically non-technical nature of the problem. In this case, the "selection of vendors" was not a technical effect and the mere "interaction" with technical elements was not enough to make the whole process technical as required by the jurisprudence.

The appellant also argued that the difference of identifying a group of vendors rather than a single vendor as in D1 implied a problem of logistics, which was not a business method. In the board's view, producing an itinerary is not technical as it involves only standard human behavioural concepts such as going to the bank and then going to the supermarket. As to the appellant's argument that the physical act of going to the locations conferred technical character on these thoughts, the board saw something of a well-known argument that could be termed the "broken technical chain fallacy" after decision T 1741/08 (see point 2.2 "Assessment of technical effect" below). This decision dealt with the fairly common situation that arises in connection with graphic user interfaces (GUIs) where a technical effect might result from the user's reaction to information. The decision essentially concluded that a chain of effects from providing information to its use in a technical process is broken by the intervention of a user. This applied to the case at issue because any possible technical effect depended on the user's reaction to the itinerary.

At the oral proceedings, the appellant stressed that the system of D1 only identified one facility whereas the invention identified a group of vendors and gave navigation information about how to visit them. In the board's view, this was another example of a standard argument, which could be termed the "non-technical prejudice fallacy". The argument essentially invokes non-technical aspects as a reason for not modifying the prior art, whereas these features cannot in fact contribute to inventive step. The question is not whether the skilled person would consider providing these features because that has already been decided in formulating the technical problem. The question is simply how it would be done. In this case, the "how" comprised conventional hardware carrying out the tasks in an obvious way.

2.1.2 Games

In T 42/10 claim 1 defined a method which, based on outcomes of games, calculated indications of the skills of the players by passing messages between nodes of a factor graph. The board had to determine to what extent the features of the claim had a technical character and could contribute to inventive step. The board referred to the decision of the Court of Appeal of England and Wales in Re Gale's Application [1991] RPC 305. Mr Gale had found an algorithm for calculating square roots, which he had implemented as a computer program. The court was concerned with the question of whether that was excluded under Section 1(2) of the Patents Act 1977. That provision gives effect, in UK law, to Art. 52(2) EPC. Lord Justice Nicholls delivered the leading judgment. He found that the program did not "embody a technical process which [existed] outside the computer", and that although the computer "will be a better computer when programmed with Mr Gale's instructions," it did not "solve a 'technical' problem lying within the computer". The board's approach to assessing questions of what is and what is not technical about a computer-implemented method was, in this case, to ask the same questions as Nicholls LJ in Re Gale's Application. The first was: what does the method as a whole do, and does it produce an overall technical result? The second was: if there is no overall technical result, does the method at least have a technical effect within the computer? If both questions were answered in the negative, no technical problem had been solved and there could be no inventive step. The board's view regarding technicality could be summarised as follows: The overall aim of keeping players interested is not technical. The intermediary aim of assessing and comparing playing performance is not technical.

The representation of performance by probability distributions, and the updating of them, are mathematical methods. The use of factor graphs with message passing is a matter of mathematics or abstract computer science. The board concluded that the only technical feature defined in this claim was the (computer) processor. The subject matter of claim 1, therefore, did not involve an inventive step if it would have been obvious to the skilled person, who had the task of implementing the method, to use a computer processor.

2.2 Assessment of technical effect

In T 1741/08 Board 3.5.06 dealt with the question of whether a technical effect could be attributed to a particular layout of a graphical user interface (GUI). It was undisputed that according to established case law of the boards of appeal, the subject-matter of a claim cannot be inventive if there is no additional technical effect over and above any technical effects present in the closest prior art. The appellant argued that the required technical effect was to be seen in the reduction of computer resources resulting from the layout of icons, which made it easier, particularly for an inexperienced user, to identify the stage reached in a process of data input requiring a number of steps and sub-steps. It was furthermore argued that any layout improvement which has the effect of "lowering the cognitive burden of the user", at least in the context of an input operation, should be at least potentially the subject of a patent.

The board did not agree with this line of argument. It found that the reduction in use of resources would be caused by the way the brain of the user perceives and processes the visual information given by a particular way of presenting information. Following T 1143/06, the board considered a GUI layout as such to be non-technical, being a "presentation of information" (Art. 52(2)(d) EPC). In the case at issue, the arrangement of icons displayed was intended to convey information, namely at which step in the input process the user found him- or herself.

The cases T 643/00, T 928/03 and T 333/95 cited by the appellant were different from the case under consideration in that there was something other than the simple choice of what information to display and with what layout to display it, so that in these exceptional cases the displayed information might play a part in the assessment of inventive step of the claimed invention.

In T 862/10 the application related to a notification system and to the positioning and rendering notification heralds based on the user's focus of attention and activity. In its catchword the board stated that the choice of where to put an object on a computer display depending on a value assigned to that object (its "urgency") could not be considered to have a further technical effect. Furthermore, the movement of the object on the display in response to a change of said value was also considered not to have a further technical effect.

The board referred to its case law (T 1143/06 and T 1741/08) that displaying a value assigned to an object by means of its relative positioning, or by moving it on the screen, was clearly a presentation of information. The particular effects of the claimed invention put forward by the appellant, "minimising information overload and distraction", could not be considered technical in nature according to the case law, being determined by psychological factors and typical to the question of how to present information in a particular context.

Determining (or attempting to determine) a user's visual focus of attention as a point on a screen and displaying objects in positions relative to that point could be considered to have a technical effect, but the particular choice of where to display an object dependent on a value assigned to that object (its "urgency") could not be.

In D1 (closest prior art) the question was considered of how to present notifications in such a way that high priority messages attracted attention; a skilled person would normally be tempted to continue along the same line, i.e. to ensure that urgent messages would receive even more attention. One way of grabbing the user's attention which would naturally come to mind is to place the urgent information in the user's focus of attention (visual or otherwise). Such attention grabbing is in fact part of human nature. For example, when a mother wants to attract the attention of her child which has totally immersed itself in a television programme, she may decide to stand in front of the television, i.e. in the child's "focus of visual attention". Leaving aside the normal physical changes that would occur as a consequence of the inherent technical nature of a computer display, e.g. changes in the intensity of various pixels, the only effect that could possibly be caused by a continuous movement of the display object is to attract the attention of the person looking at the display and present information to him or her that a certain message is urgent. It may also imply a kind of time limit for reacting (before the object reaches the focus of attention).

However this is also only a presentation of information; the application did not include any disclosure of any technical consequences of reacting or not within this implied time limit. In other words, continuously moving the display object could serve no other objective purpose than that of presenting information as such. It therefore produced no further technical effect (i.e. no technical effect apart from the normal physical changes which inherently take place in a computer display) and did not contribute to the presence of an inventive step.

In T 1539/09 the board stated in its catchword that the activity of programming – in the sense of writing code – is a mental act, at least to the extent that it is not used in an actual application or environment to produce a technical effect. Therefore the definition and provision of a programming language does not per se contribute to a technical solution, even if the selection of a programming language helps to reduce the programmer's intellectual effort.

The invention related to a graphic programming language and environment that was designed to allow a user to write code without much training or expertise. The board took the view that reducing a programmer's intellectual effort was not a technical effect. This applied all the more since the effect was likewise sought for all programs developed, regardless of their purpose.

3. Features not contributing to the solution of the problem

In T 1009/12 the board stated in its catchword: "An ineffective concentration of a compound is considered as an arbitrary feature not contributing to the solution of the underlying problem and therefore not further considered".

The board referred to the established case law, according to which features that do not contribute to the solution of the problem set in the description are not considered in assessing the inventive step of a combination of features (T 206/91). Since process claim 1 included silver plating compositions including unspecified, therefore also ineffective concentrations of the nitro aromatic oxidant this feature could only be considered as an arbitrary feature, because it was not credible that it contributed to the solution of the underlying technical problem.

The board therefore did not further consider it. Moreover, it considered that for the same reason, the formulation of a less ambitious problem, i.e. to provide an alternative solution with the same effect, based on this alleged distinguishing feature could not hold either. Since there were no further distinguishing features it was not feasible to identify the technical problem to be solved. Since no technical problem solved by the subject-matter of process claim 1 could be identified, the board considered that claim 1 of the first auxiliary request lacked inventive step.

4. Chemical inventions – synergistic effect

In T 1814/11 the problem to be solved was to provide an alternative synergistically active fungicidal composition based on prothioconazole. The composition consisting of prothioconazole and picoxystrobin claimed in claim 1 of the patent in suit was proposed as a solution. The board noted that based on document 1 (closest prior art), the skilled person would have had no reason to mix prothioconazole with the picoxystrobin mentioned in document 2. Synergistic effects were not foreseeable, i.e. even if a combination of two specific compositions had a synergistic effect as in document 1, such synergy could not necessary be expected if the structure of one of the two compositions were modified. The respondent argued that it would have been obvious for the skilled person to experiment with compositions from the same class, i.e. strobilurine, and arrive at the claimed mixture through trial and error. This argument failed to convince the board. It was true that fungicidal compositions that belong to the same class usually have either similar mechanisms of action and/or possibly similar chemical structures. This did not mean, however, that if some fungicidal compositions from the same class worked synergistically in combination with a specific fungicidal composition, all or the majority of compositions from that class would also work synergistically with the specific composition. Synergy was not in principle foreseeable and therefore could not be attributed to a specific mechanism of action and/or structure. On the basis of document 1 and without knowing the invention, the trial-and-error experimentation referred to by the respondent would in this case amount to testing mixtures of different fungicidal compositions with prothioconazole, without it being possible for the skilled person to foresee whether at least one mixture actually had a synergistic effect.

5. Obvious desiderata

In T 661/09 the application related to a reflective polarizer that transmits light of one polarization and reflects light of another polarization.

The only technical feature defining the claimed reflective polarizer in terms of structural limitations was that the polarizer was a "multilayer reflective polarizer", meaning that the reflective polarizer comprised multiple layers. Rather than specifying the concrete nature of the multiple layers by indicating, for instance, their total number, individual thicknesses, composition or sequence in a stack of layers, the remaining features of claim 1 only gave numerical values for particular spectral reflection properties. These properties of the polarizer were the result of functional interactions between individual layers, each layer requiring specific layer characteristics.

Therefore, the only features distinguishing the claimed device from the closest prior art expressed no more than a set of desiderata, without any indication of a causal link between the desired properties and the constitution of the claimed device. Insofar as the claim did not define any concrete measures on how to ensure that the claimed properties were effectively obtained, the claimed properties remained at an abstract or conceptual level. Accordingly, the issue of inventive step boiled down to the question of whether or not the skilled person, in view of the available prior art and his/her common general knowledge, would in an obvious way have envisaged the claimed set of desiderata.

The board concluded that the features actually claimed merely expressed obvious desiderata at an abstract level.

II. PATENT APPLICATION AND AMENDMENTS

A. Claims

In T 459/09, the appellant (patent proprietor) filed an amended set of claims in the appeal proceedings. The board held that claim 1 of each of the requests lacked clarity.

The board noted that objections under Art. 84 EPC cannot be raised against a granted patent, even though they are manifest. However, the situation was different in the case of the maintenance of a patent in amended form.

According to the jurisprudence of the boards of appeal, there were limits to the competence accorded by Art. 101(3) EPC as regards the examination of an amended patent. However, the jurisprudence of the boards also comprised examples in which the combination of claims of a granted patent were considered to constitute substantial amendments which justified an examination under Art. 84 EPC.

The board judged as follows:

An amendment consisting of the incorporation of a technically meaningful feature in an independent claim of a granted patent does indeed represent an attempt to overcome an objection within the framework of Art. 100 EPC against the patent as granted, the amendment having to be occasioned by a ground for opposition (R. 80 EPC). It follows that such an amendment is of a substantial nature and will normally have an effect on the substantive examination, such as for example on the assessment of novelty and inventive step.

Any amendment that can be qualified as being of a substantial nature in the above sense would in principle justify an unrestricted exercise of the examination power derivable from Art. 101(3) EPC, irrespective of the kind of amendment performed. Specifically, it is immaterial whether the amendment arises from the combination of a feature from the description with an independent claim, or from the literal combination of claims of the granted patent.

The amended patent would thus normally be examined pursuant to Art. 101(3) EPC so as to establish whether it meets all the requirements of the EPC. A deviation from this rule may, however, not be excluded in particular cases. This had to be judged on a case-by-case basis.

In accordance with Art. 112(1)(a) EPC, Technical Board of Appeal 3.2.08 has referred points of law to the Enlarged Board of Appeal with interlocutory decision of 2 April 2014 in case T 373/12. See Part I, point 3.1.1, with reference to G 3/14.

B. Unity of invention

1. Euro-PCT applications – consideration of unity by the EPO

Decisions T 1981/12, T 2473/12 and T 2459/12 are concerned primarily with the interpretation and application of R. 164 EPC "Consideration of unity" by the EPO. It is to be noted, however, that, by decision of the Administrative Council of 16 October 2013 (CA/D 17/13), OJ EPO 2013, 503, this provision has been amended and renamed as R. 164 EPC "Unity of invention and further searches" with effect from 1 November 2014.

In each of the cited cases, the international search had been carried out by an International Searching Authority other than the EPO and all claims had been searched. However, in the European phase the application was found to lack unity. Therefore the supplementary European search report was drawn up only for the invention first mentioned in the claims (R. 164(1) EPC) and the applicant was invited to limit the application accordingly (R. 164(2) EPC). In addition, an objection was raised in respect of unsearched subject-matter (R. 137(5) EPC, formerly R. 137(4) EPC). The applicant failed to comply with the aforementioned invitation and ultimately the application was refused (Art. 97(2) EPC).

The main conclusions of the board in T 1981/12, as set out in the catchword to the decision, are as follows:

Claims whose subject-matter has not been covered by a search report drawn up by the EPO will not be examined by the EPO for novelty or inventive step.

Where subject-matter is covered by a search report drawn up by an ISA other than the EPO in the international phase but is not covered by the supplementary search report drawn up by the EPO in the European phase in the application of R. 164(1) EPC, the subject-matter is not to be regarded as covered by a search report for the purposes of R. 164(2) EPC.

Where on entering the European phase the application documents are considered by the EPO not to meet the requirements of unity, with the result that a supplementary European search report is drawn up on only those parts of the application which relate to the invention first mentioned in the claims (R. 164(1) EPC), the applicant is not entitled to have a further search report drawn up to cover the other invention or inventions.

This decision was applied in T 2473/12. The catchword also stated:

The effect of R. 164(2) EPC whereby a national of a non-EPC contracting state may be obliged to file one or more divisional applications in order to obtain protection for subject-matter not covered by the supplementary European search report does not amount to different national treatment within the meaning of Art. 2(1) of the Paris Convention.

Concerning the application of R. 164(2) EPC, decision T 2459/12 confirmed the approach set out in the Guidelines for Examination (see catchword).

However, with regard to R. 137(5) EPC, while the board in T 1981/12 had doubted its appropriateness as a basis for refusing an application based on the requests in that case, the present board concluded (see catchword):

Where, following the drawing up of a supplementary European search report by the EPO, an applicant files amended claims seeking protection for subject-matter which is not covered by the supplementary European search report as a result of the application of R. 164(1) EPC, an objection under R. 137(5) EPC should be raised. Non-compliance with R. 137(5) EPC is a ground for refusing an application.

C. Sufficiency of disclosure

1. Clarity and completeness of disclosure

According to T 593/09, where a claim contains an ill-defined ("unclear", "ambiguous") parameter and where, as a consequence, the skilled person would not know whether he was working within or outside of the scope of the claim, this, by itself, is not a reason to deny sufficiency of disclosure as required by Art. 83 EPC. Nor is such a lack of clear definition necessarily a matter for objection under Art. 84 EPC only. What is decisive is whether the parameter is so ill-defined that the skilled person is not able, on the basis of the disclosure as a whole and using his common general knowledge, to identify (without undue burden) the technical measures (e.g. selection of suitable compounds) necessary to solve the problem underlying the patent at issue (see also point 4.2 "Article 83 and the clarity of claims" below).

2. Reproducibility

In T 432/10 the board of appeal found that there is insufficiency of disclosure if the skilled person, taking into account the entire teaching of the patent, is not able to rework an invention which is defined in the claims in a completely clear and comprehensible manner, unless he disregards a meaningful feature thereof. The invention, which concerned a non-therapeutic process for improving the glucose metabolism of a companion animal, referred to both a one-time administration and a diet. As a consequence, the board found that the claimed invention was contradictory in itself, so that the skilled person was at a complete loss as to what he should do.

The opposition division had argued that the feature "maintaining said animal on said diet ..." did not set a definite time limit for the use of the composition. The board, however, concluded that this interpretation would be contrary to the interest of legal certainty, as it would mean that a feature that was perfectly understandable per se would have to be deliberately ignored in order to make the invention workable. Such a step had to be distinguished from a situation concerning clarity, in which an unclear term figuring in a claim is interpreted in the light of the description. There, the unclear feature is not simply ignored. In the case at issue, however, the mental deletion of the feature in question would mean switching from the unworkable and thus insufficiently disclosed invention to a different invention that was workable but not claimed. The board found that the requirements of Art. 83 EPC 1973 were not met.

2.1 Reproducibility without undue burden

2.1.1 Trial and error

The invention at issue in T 852/09 concerned the use of an enhancer in the manufacture of a composition for lowering cholesterol levels. The enhancer was not defined by any specific structural features, but merely by a functional feature and the capability to lower the level of LDL cholesterol. Consequently, the claim concerned all relevant chemical compounds without any restrictions to compound classes or chemical structure.

The application did not provide the skilled person with any guidance enabling her/him, without undue experimentation, to identify the enhancers as defined in the claim. The board found that, in a search for the enhancers, the skilled person would have to test a virtually unlimited amount of chemical compounds of various chemical structures.

Therefore, following T 1063/06, the board held that since the enhancers to be used were characterised in functional terms only and the claim merely represented for the skilled person an invitation to perform a research programme, he/she could not carry out the invention within the entire scope claimed without undue burden (see also T 155/08). The requirements of Art. 83 EPC were therefore not met.

2.2 Post-published documents

The board in T 1273/09 confirmed that sufficiency of disclosure must be satisfied at the effective date of the patent, i.e. on the basis of the information in the patent application together with, or on the basis of, the common general knowledge then available to the skilled person. Therefore, the disclosure in post-published documents can only be taken into account for the question of sufficiency of disclosure if it was used to back up the positive findings in relation to the disclosure in a patent application (see e.g. T 609/02).

3. The requirement of sufficiency of disclosure in the biotechnology field

3.1 Requirements relating to nucleotide and amino acid sequences

Under R. 30(3) EPC, where the applicant has not filed the necessary sequence listing prescribed in R. 30(1) EPC, the EPO shall invite him to furnish it on payment of a fee. In J 7/11, the Legal Board held that such invitations cannot be given orally only – a telephone call was, given the short time available, useful, but had to be followed by a written invitation enumerating all objections raised. Failure to do so amounted to a substantial procedural violation.

In J 8/11 the crucial issue on appeal was the interpretation of the term "disclosed" in R. 30(1) EPC, namely the question whether a patent application which related to the use of polypeptides well known in the prior art and which identified these polypeptides by their common names and by database accession numbers concerning specific representative sequences had to be regarded as "disclosing" amino acid sequences.

The Legal Board pointed out that when new R. 27a EPC 1973 (R. 30 EPC) entered into force, it was not the legislator's intention to make sequence listings mandatory in situations where the description merely referred to prior-art sequences, e.g. by indicating their common names and database accession numbers. The term "disclosed" in R. 27a EPC 1973 was thus interpreted in a narrow manner in line with the explanatory memorandum in CA/7/92 and the established principles concerning the deposit of biological material.

On revision of the EPC 2000, the travaux préparatoires did not contain any indication that the legislator intended this paragraph to be understood differently from before. However, in the Notice from the EPO dated 12 July 2007 concerning the filing of sequence listings (Special edition No. 3 of OJ EPO 2007, 84, C.2), the EPO interpreted R. 30(1) EPC not exactly in the same way as it had done before, making a significant distinction with respect to prior-art sequences referred to in the application for which the wording of R. 30(1) EPC did not provide any basis. Furthermore, some criteria used as a basis for the distinction required a technical evaluation of the application, which did not fall within the competence of the Receiving Section. The Receiving Section is restricted to a merely formal examination of the sequence listing requirements (see J 7/11).

The Legal Board concluded that prior-art sequences do not require the filing of a sequence listing and that the Receiving Section had been wrong to apply R. 30 EPC.

4. The relationship between Article 83 and Article 84 EPC

4.1 Article 83 EPC and support from the description

In T 553/11 the board pointed out that if the proprietor wishes to argue for a narrow scope of a claim, this should be on the basis of the ordinary wording of the claim, and not on the basis of something appearing only in the description (following T 1404/05). The board also referred to T 681/01, where it was emphasized that the normal rule of claim construction is that the terms used in a claim should be given their ordinary meaning in the context of the claim in which they appear. The description may not be used to rewrite the claim and redefine the technical features required by the claim in a way not warranted by the wording of the claim itself. In particular the description cannot be relied on to exclude subject-matter from the claim which the ordinary meaning of the terms used would include as part of what is claimed.

4.2 Article 83 EPC and clarity of claims

According to T 593/09 the requirement of a sufficient or "enabling" disclosure within the meaning of Art. 83 EPC is different from and independent from the clarity requirement pursuant to Art. 84 EPC, namely that the claims, which define the matter for which protection is sought, "shall be clear and concise". The board saw this distinction as underlying the decision in T 1062/98, where it was held that whether the skilled person is capable of determining whether a certain feature would be infringing a feature claimed is not a matter of sufficiency of disclosure as required by Art. 83 EPC, but a matter to be decided by the national courts. The board in T 1062/98 had further held that the determination of the scope of the claim in fact relates to the question whether the claims properly define the matter for which protection is sought, which are the requirements imposed by Art. 84 EPC and R. 29(1) EPC, which do not form grounds of opposition.

According to the board in T 593/09 there is thus a distinction between the meaning of "clear" in Art. 83 EPC, which concerns the disclosure (the "technical teaching") of the application or the patent on the one hand, and in Art. 84 EPC, where that expression relates to the claims, which "shall define the matter for which protection is sought" on the other hand. In short, there is a distinction between clarity of what has been disclosed and clarity of what is claimed. This distinction is, however, not always properly made, in particular in respect of so-called "ambiguous parameters".

The board in T 1526/09 observed that it had been decided in T 593/09 that a claim containing a vague or ambiguous parameter preventing the skilled person from knowing whether he was working within or outside the scope of the claim did not result in insufficient disclosure of the invention. The crucial question in deciding on sufficiency of disclosure was whether the parameter was so vaguely defined that the skilled person could not identify, in the patent as a whole, the measures required to solve the underlying problem.

It observed that the board hearing the appeal in T 593/09, in which the inadequately defined parameter had been a particular crystallisation temperature of a source material, had considered that temperature essential to solving the underlying problem, i.e. the parameter at issue had been essential to preparing the product. In the case now to be decided, however, reduced chargeability was an advantageous property of the product obtained by a process disclosed in the patent in suit. The vague definition of chargeability affected the clarity of the claimed subject-matter but did not prevent the skilled person from preparing the claimed product. Since lack of clarity was not a ground for opposition under Art. 100 EPC, the grounds invoking Art. 100(b) EPC could not preclude maintenance of the patent on the basis of the main request.

D. Priority

1. Enabling disclosure in the priority document

According to the board in T 107/09, the antibody MR1 was indispensable in order to reproduce the invention of claim 1. The "written" disclosure in the earlier US application from which priority was claimed, even if supplemented by common general knowledge, would not enable the skilled person to carry out the invention. The hybridoma cell line producing the antibody MR1 had been deposited with the ATCC (American Type Culture Collection) only after the filing date of the earlier application. In view of the absence of explicit provisions in the EPC as to when a deposit of biological material had to be made in relation to an earlier application (R. 28 EPC 1973 being concerned with European applications), the board referred to the case law of the Enlarged Board of Appeal, according to which the requirement of sufficiency of disclosure had to be complied with – in relation to an earlier application from which priority was claimed – at the date of filing of that application (cf. G 2/93, OJ EPO 1995, 275, and especially G 1/03, OJ EPO 2004, 413; cf. in this connection also Notice of 18 July 1986, OJ EPO 1986, 269, and now Notice of 7 July 2010, OJ EPO 2010, 498). Thus, the board reasoned, if the deposit of biological material was necessary for the requirement of sufficiency of disclosure to be fulfilled for a "priority application", the deposit of this material had to have been made no later than the date of filing of that earlier application. This was not the case here.

The board also rejected the appellant's argument that since a deposit in relation to a US application was not necessary already at its date of filing, the US application at issue here complied with the requirements of US patent law and consequently that it must be considered a regular national filing (Art. 87(2) EPC 1973) and thus give a right to priority. What Art. 87(2) and (3) EPC merely set out was that the date of filing of an application that may give rise to a right to priority under the EPC is accorded in accordance with national law for the purposes of Art. 87(1) EPC. It could not be inferred from these provisions that the standards of national law are applied in relation to other requirements of a potential priority application, for example, in relation to criteria for determining the disclosure content of such an application.

In sum, the earlier US application did not provide a disclosure sufficient for the skilled person to carry out the invention in claim 1. Therefore, the requirement of the "same invention" according to Art. 87(1) EPC could not be considered as being fulfilled and hence claim 1 could not enjoy the right of priority from the earlier application.

2. First application – partial and multiple priorities

In case T 1222/11, claim 1 had been amended to include six disclaimers for the purpose of restoring novelty over D4b, an earlier international (Euro-PCT) application of the same applicant, which had been published within the priority interval (cf. Art. 54(3) EPC). These disclaimers corresponded to six compositions disclosed in D4b and falling within the definition of the composition defined in positive terms in claim 1. Having regard to the criteria of G 1/03 (OJ EPO 2004, 413) for the allowability of undisclosed disclaimers, the board pointed out that it was necessary to determine whether D4b was prior art pursuant to Art. 54(3) or 54(2) EPC and therefore whether the claimed priority was valid.

It found that the combination of features defined in positive terms in claim 1 (i.e. without any disclaimer) could not be distinguished from the disclosure of earlier application D4b, and was not entitled to the priority claimed as far as it concerned the subject-matter already disclosed in D4b (see Art. 87(1), (4) EPC). The compositions of D4b were therefore prior art under Art. 54(2) EPC. As the anticipatory disclosure of D4b was not an accidental anticipation (see G 1/03), the disclaimers based on it were not allowable under Art. 123(2) EPC.

In connection with the priority question, the board set out some additional remarks relating to the principles of claiming multiple priorities and partial priority (see Art. 4F Paris Convention; Art. 88(2), (3) EPC; G 2/98, OJ EPO 2001, 413; T 15/01).

In G 2/98 (point 6.7 of the Reasons), the Enlarged Board had stated in the context of the "OR" claim situation: "The use of a generic term or formula in a claim for which multiple priorities are claimed in accordance with Art. 88(2), second sentence, EPC is perfectly acceptable under Art. 87(1) and 88(3) EPC, provided that it gives rise to the claiming of a limited number of clearly defined alternative subject-matters." In the opinion of the board, the condition "provided that it gives rise to the claiming of a limited number of clearly defined alternative subject-matters" should be given a different meaning from that attributed to it in decisions T 1877/08, T 476/09, T 1443/05 and T 1127/00. It could not be meant to set out the manner in which the subject-matter of the "OR" claim must be defined. This would, at least in relation to generic terms, be at variance with the disclosure test based on the principle of an unambiguous and direct disclosure (see G 3/89, OJ EPO 1993, 117).

For the purpose of the assessment required by Art. 88(3) EPC, the board considered that the wording of the condition referred to the ability to conceptually identify, by comparison of the claimed subject-matter of the "OR" claim with the disclosure of the multiple priority documents, a limited number of clearly defined alternative subject-matters to which the multiple rights of priority claimed could be attributed or not (see examples in the Memorandum drawn up by FICPI (M/48/I, Section C) for the Munich Diplomatic Conference in 1973, as cited in G 2/98). That this comparison should give rise to a limited number of clearly defined alternative subject-matters was necessary in order to identify which parts of the claims benefited from the effect of the priority right defined in Art. 89 EPC. The aforementioned condition of G 2/98 should also apply when assessing partial priority in relation to a single priority document.

The board added that it was not the task of the EPO to determine ex officio to which parts of an "OR" claim could be attributed the right(s) of priority claimed. In relation to intermediate prior art, the burden of proof that the "OR" claim benefited from a partial priority, if prima facie not immediately obvious, was on the person alleging the existence of the priority.

E. Amendments

1. Article 123(2) EPC

1.1 General issues

The board in T 2284/09 stated that, with respect to the amount of acid to be added, as denoted in the application as filed by the differing meanings of the terms "free" and "excess" acid, it emerged that there was no disclosure in the application as filed of a process whereby a single, numerically fixed amount of acid was added to the unneutralised polyether polyol.

From the difference in wording of claim 1 of the main request as compared to the disclosure of the application as originally filed it emerged that the absence of the functional or relative limitation of the application as filed from the definition of the amount of acid to be added had the consequence that claim 1 of the main request defined a fundamentally different process from that specified in the application as originally filed. The arguments of the appellant (patent proprietor) that the skilled person would consult the description in order to ascertain what the claim was intended to specify presupposed that there was a reason to consult the description, e.g. some prima facie unclarity or inconsistency in the claim. This was, however, not the case. On the contrary, the claim of the patent as granted provided a coherent, cogent technical teaching and did not as it stood present the reader with any aspect that would indicate a need for interpretation or clarification. Nor had the appellant explained in what manner the wording of the claim would be deficient and therefore in need of interpretation. The board concluded that claim 1 of the main request did not meet the requirements of Art. 123(2) EPC.

1.1.1 Notion of content of the application

In T 667/08 regarding procedural matters, at the appellant's request received on 4 January 2011, the proceedings were stayed pending the outcome of G 2/10 (OJ EPO 2012, 376). They were then resumed after the Enlarged Board had decided on the referral. The question arose whether a sufficient basis existed in the original disclosure to allow a generalisation. It is an undisputed principle in the jurisprudence of the boards of appeal of the EPO that an amendment is allowable under Art. 123(2) EPC if the subject-matter resulting from the amendment is directly and unambiguously derivable from the original application documents, i.e. the description, the claims and the drawings, using common general knowledge.

It is not necessary that the subject-matter resulting from the amendment be explicitly disclosed in the original application. The board stated that it was therefore essential, when deciding on issues of added subject-matter, to identify the actual teaching conveyed by the original disclosure, i.e. the technical information that the skilled person reading the original disclosure would have derived from its content (description, claims and drawings) considered in its entirety. This approach might lead to the identification of subject-matter which was not explicitly revealed as such in the application as filed, but nevertheless is derived directly and unambiguously from its content. Literal support is not required by the wording of Art. 123(2) EPC (see point 4.1.4 of the Reasons).

The board in T 2619/11 took the view that the focus of the first-instance decision was disproportionately directed to the structure of the claims as filed to the detriment of what was really disclosed to the skilled person by the documents as filed. The application was directed to a technical audience rather than to a philologist or logician, for which audience an attempt to derive information from the structure of dependent claims would lead to an artificial result.

1.1.2 Value ranges, lists, singling out, generalisation

When it came to examining the requirements under Art. 123(2) EPC in T 99/09, the appellant/patent proprietor maintained that "immediate release" in claim 1 was to be regarded as an invisible term without any additional technical content. The opposition division had revoked the patent but on this point had considered that "immediate release" did not add technical content to the claim as long as the claim also comprised a specific dissolution profile. The board observed that this term had been disclosed in the application as filed only in combination with other structural or functional features.

It was always disclosed in combination with technical features, never in isolation; since these were absent from the subject-matter of claim 1 of the main request, it was a generalisation. The application as filed did not provide a basis for such a generalisation, i.e. for any type of "immediate release" composition having a dissolution profile as claimed. Moreover, the board did not agree that the "immediate release" feature was synonymous with the claimed dissolution profile: this term had a generally established technical meaning in galenic pharmacy. Finally, after detailed examination, the board concluded that "immediate release", although vague, should therefore be regarded as a functional feature, and that the term in question made a technical contribution to the claimed subject-matter and could not be regarded as "invisible" (in breach of Art. 123(2) EPC).

1.2 Intermediate generalisation – non-disclosed combinations

In T 1906/11 the board highlighted that whether an amendment constitutes an intermediate generalisation is irrelevant for the purposes of assessing its admissibility. According to that decision, the only relevant question would be whether a skilled person faced with the amended version of the application or patent, as compared to a skilled person having seen only the version originally disclosed, would derive from that amended version any additional technically relevant information. Only if such additional information was derivable can there be an infringement of Art. 123(2) EPC (point 4.2 of the Reasons). In T 248/12 the board stated that in respect of the "technical relevance" of the added information, case T 1906/11 could not be understood to define a new standard for judging amendments with respect to Art. 123(2) EPC since this would be at odds with the "gold standard".

The board emphasised that "relevance", as such, of the technical information was not of importance for deciding upon the issue of Art. 123(2) EPC, rather simply whether this technical information received by the skilled person is new having regard to the content of the originally filed application.

In T 500/11, the amendment in claim 1 at issue corresponded to the restriction of the range "between 50 to 10 000 ppm of chromium" by the lower end value of 550 ppm. It was uncontested that there was a literal basis for this specific value in example 3 of the patent in suit, however not in combination with the other features of amended claim 1 under dispute. Appellant II (opponent) argued that the amendment consisted of an inadmissible intermediate generalisation in the sense of T 962/98, because the feature 550 ppm Cr was inextricably linked to the other features of the process according to example 3 and thus this specific value could not be arbitrarily extracted from its context. The question to be answered now was whether the feature "550 ppm Cr" was inextricably linked to the other features defined in example 3 of the patent in suit. The board observed that all the features were parameters which might be varied individually and each of these variations would necessarily lead to a different result on the selectivity to TCS. If the feature "550 ppm chromium" was inextricably linked to specific other parameters of example 3, this would mean that the selectivity to TCS would only be achieved in the definite combination of "550 ppm chromium" with the specific other parameters of example 3.

This, however, was manifestly not the case here. In the case at issue the aim of the invention – namely increasing the selectivity to TCS – was merely achieved by the addition of selected amounts of chromium to silicon and thus this feature was not "closely related to the other characteristics of the working example and applies directly and unambiguously to the more general context" as required in T 962/98. The current decision was also in line with T 273/10. It followed that the picking out of the value 550 ppm from the working example 3 was therefore plainly acceptable in the case at issue, with the consequence that the requirements of Art. 123(2) EPC were fulfilled.

1.3 Technical contribution – addition or deletion of a feature

In T 248/12 the board emphasised that the standard whereby an amendment must be directly and unambiguously derivable, using common general knowledge, and seen objectively and relative to the date of filing, from the whole of the application as filed, remains a pre-requisite for judging any amendment with respect to the requirement of Art. 123(2) EPC. Hence, the "relevance", as such, of the technical information is not of importance for deciding upon the issue of Art. 123(2) EPC. In the case at issue it could also not be accepted by the board that when a skilled person applied his common general knowledge, he would then arrive at the combination of features in claim 11 of auxiliary request 2.

Nothing had been filed by the appellant (proprietor) to support this argument, nor was any such information derivable from the application as filed. The appellant's (proprietor's) argument that a skilled person would realise what elements were technically relevant to the invention when adding certain further structural features into the claim was entirely subjective.

It thus followed that the introduction into claim 11 of both the first and second features resulted in the skilled person being presented with a new combination of features (i.e. new technical information) which he would not derive directly and unambiguously, using common general knowledge, from the application as filed. Claim 11 thus contravened Art. 123(2) EPC and auxiliary request 2 was therefore not allowable.

1.4 Disclaimers

1.4.1 Applicable law – decisions G 1/03, G 2/03 and G 2/10

The Enlarged Board in G 2/10 (OJ EPO 2012, 376) answered the questions referred to it by interlocutory decision in T 1068/07 (OJ EPO 2011, 256) as follows:

1a. An amendment to a claim by the introduction of a disclaimer disclaiming from it subject-matter disclosed in the application as filed infringes Art. 123(2) EPC if the subject-matter remaining in the claim after the introduction of the disclaimer is not, be it explicitly or implicitly, directly and unambiguously disclosed to the skilled person using common general knowledge, in the application as filed.

1b. Determining whether or not that is the case requires a technical assessment of the overall technical circumstances of the individual case under consideration, taking into account the nature and extent of the disclosure in the application as filed, the nature and extent of the disclaimed subject-matter and its relationship with the subject-matter remaining in the claim after the amendment.

Case T 1870/08 related to an undisclosed disclaimer. Decision G 2/10 was therefore, prima facie, not concerned with the same situation, so it was questioned by the board whether that decision was applicable at all to the case before it.

It concluded, after detailed reasoning, that the test of G 2/10 (see G 2/10, part 1a of the Order, points 4.5.1 and 4.5.2 of the Reasons) also applies to an amendment involving an undisclosed disclaimer by means of which a claim has been rendered new over a European patent application according to Art. 54(3) EPC. The examination of the admissibility of the amendment for compliance with Art. 123(2) EPC must be made separately for the disclaimer per se and for the subject-matter remaining in the claim (see point 4. of the Reasons).

In decision G 2/10 the Enlarged Board of Appeal did not consider that decision G 1/03 (OJ EPO 2004, 413) was exhaustive as to the conditions that needed to be fulfilled for an amendment consisting of the introduction of an undisclosed disclaimer to be regarded as allowable under Art. 123(2) EPC. The board in T 2464/10 interpreted these remarks as an instruction to apply the further test developed in this decision, in addition to those set out in G 1/03, in order to carry out a full assessment of whether an undisclosed disclaimer meets the requirements of Art. 123(2) EPC. The further test to be applied is whether the skilled person would, using common knowledge, regard the remaining claimed subject-matter as explicitly or implicitly, but directly and unambiguously, disclosed in the application as filed (see also T 748/09).

1.4.2 Decisions applying the criteria established by G 1/03, G 2/03 and G 2/10

a) Drafting of disclaimers

In T 1843/09 (OJ EPO 2013, 502), the opponent argued that the disclaimer contravened Art. 123(2) EPC because the wording of the disclaimer that the claimed film "is other than a film of Comparative Example 4 of EP-A 0546184" did not represent a technical feature. The board did not accept this argument.

Although it was true that technical information could not directly be extracted from the wording of the disclaimer in claim 1 as such, it should be noted that the disclaimer did not merely cite a published patent document, but clearly referred to a specific disclosure in D15, namely a single film described in comparative example 4. Table 4 of D15 characterised this film unambiguously by a number of technical features. The skilled person was therefore able to determine simply by reading the comparative example in D15 which technical embodiment should be excluded from the scope of the claim. The disclaimer in claim 1 therefore represented a negative technical feature in the sense of G 1/03.

In T 1836/10 the examination division had based its refusal of the application on Art. 53(a) in conjunction with R. 28(c) EPC, according to which biotechnological inventions using human embryos for industrial or commercial purposes are excluded from patentability. It was not disputed that the negative feature introduced could not be derived from the application as filed.

The appellant justified the amendment of claim 1 on the basis of decisions G 1/03 und G 2/03 (OJ EPO 2004, 448), according to which an undisclosed negative feature, a "disclaimer", could be admissibly introduced to a claim if it removed subject-matter which was non-patentable under Art. 52 to 57 EPC for non-technical reasons (point 2.4 of the Reasons). In deciding whether it was admissible under Art. 123(2) EPC to introduce the disclaimer to claim 1, there was the issue of whether the disclaimer limited the patent protection sought by disclaiming subject-matter excluded from patentability under Art. 53(a) in conjunction with R. 28(c) EPC from claim 1.

To answer this question, the subject-matter of claim 1 both prior to the introduction of the disclaimer and as defined by the disclaimer were established by interpretation. The board found that the subject-matter of the claim was not limited in any way by the introduction of the disclaimer because it served to disclaim something not comprised by the claim in the first place.

The Enlarged Board of Appeal had held in its decisions G 1/03 and G 2/03 that an applicant could not amend his claims at will, and that any disclaimer needed should not exclude any more than was necessary to disclaim subject-matter excluded from patentability for non-technical reasons. Otherwise the disclaimer was to be regarded as inadmissible under Art. 123(2) EPC. In the board's view, the same applied to a disclaimer which served to disclaim subject-matter that was not comprised by the claim in the first place. The amendment of a claim by introducing such a disclaimer did not meet the intended purpose and was therefore to be regarded as inadmissible under Art. 123(2) EPC. Moreover, the claim amended by the introduction of the disclaimer did not meet the clarity requirement of Art. 84 EPC. For these reasons, the board held that the amendment to claim 1 infringed Art. 123(2) EPC.

b) Drafting of disclaimers and clarity

In T 1695/07, claims 1 to 8 of the main request were found by the board to be directed to a method for treatment of the human body by surgery which is excepted from patentability under Art. 53(c) EPC. Claim 1 of auxiliary request 2 included the feature "wherein the process is not a method for treatment of the human or animal body by surgery", i.e. a disclaimer.

Concerning the admissibility of a disclaimer excluding subject-matter not eligible for patent protection, the board in this case first stressed that, with reference to G 1/03 (OJ EPO 2004, 413), the requirements of Art. 84 EPC are also applicable to claims containing disclaimers. A clear delimitation and distinction between excepted surgical applications and possibly allowable non-surgical applications of the claimed process requires that the two methods be distinct, i.e. separable, which means that they must be of a different nature and may be carried out in different ways. In the board's view, it could not be seen how the claimed process would work without the surgical steps involved. The board concluded that the requirement of clarity was not met in the case at issue and that auxiliary request 2 was therefore not allowable.

In T 1487/09, independent claim 29 of the main request comprised a step encompassing embodiments excluded from patentability as methods for treatment of the human or animal body by surgery and therapy pursuant to Art. 53(c) EPC. The main request could not be allowed as it did not comply with Art. 53(c) EPC. Compared to claim 29 of the main request, claim 29 of auxiliary request I comprised a disclaimer aiming at excluding uses that "comprise or encompass an invasive step representing a substantial physical intervention on the body of a human or an animal which requires professional medical expertise to be carried out and which entails a substantial health risk even when carried out with the required professional care and expertise." All the requirements of the EPC have to be considered when examining a disclaimer, in particular those of Art. 84 EPC. The board observed that the uses excluded were not explicitly defined, but rather must be derived from a condition which was to be fulfilled. Whether this condition was fulfilled or not would have to be established by the reader of the claim.

Leaving room for such an assessment by the reader inevitably introduced uncertainty as to the matter for which protection was sought. Hence a lack of clarity arose, which was in breach of Art. 84 EPC. Further, the fact that the wording chosen for the disclaimer was the same as the one used in decision G 1/07 for describing a surgical method did not mean that the claim fulfilled the clarity requirements of Art. 84 EPC. This had to be handled on a case-by-case basis. A similar situation may occur when a disclaimer aiming at establishing novelty over subject-matter of a conflicting application under Art. 54(3) EPC is to be introduced.

1.5 Disclosure in drawings

If drawings are originally filed in colour at the date of filing of an application, then the technical content of these original colour drawings should be determined taking into account the available evidence when establishing the content of the application as filed, for the purpose of examining compliance of amendments with Art. 123(2) EPC (T 1544/08, points 4.4 and 4.5 of the Reasons).

1.6 "Tests" for assessing the allowability of an amendment

1.6.1 Direct and unambiguous deducibility of amendments from the application as filed

In T 612/09 Claim 1 was drawn up in the so-called Swiss-type format and concerned a dosage regime for treating a bacterial infection with the antibiotic daptomycin. According to the decision under appeal neither the dose range from "3 to 10 mg/kg of daptomycin" nor the dosage interval of "once every 48 hours" in conjunction with a dose of "3 to 10 mg/kg of daptomycin" could be derived directly and unambiguously from the application as filed.

The application as filed disclosed the administration of daptomycin to a human patient in a dose of 3 to 12 mg/kg every 24 to 48 hours.

According to established case law of the boards of appeal the range "every 24 to 48 hours" is a direct and unambiguous disclosure of the two specifically named end points, i.e. "every 24 hours" and "every 48 hours". The question to be addressed was whether the skilled person would have understood that the upper endpoint of the dosing interval, i.e. "every 48 hours", applied also to a dose of 3 to 10 mg/kg of daptomycin. The board concluded that the application as filed disclosed a functional relationship between the dosing interval and the dose of daptomycin both in terms of efficacy in treating the bacterial infection and in terms of safety as regards the problem of skeletal muscle toxicity caused by the antibiotic. In the absence of any disclosure in the application as filed that a dose of 3 to 10 mg/kg of daptomycin was not only safe but also efficacious to treat a bacterial infection in a human patient if only administered once every 48 hours, the board saw no sound reason to accept the patent proprietor/appellant's assertion that the skilled person would "unequivocally and instantly recognise" that the dosage interval of "every 48 hours" disclosed as the endpoint of a dosage interval for a dose range of 3 to 12 mg/kg daptomycin must necessarily also be applicable to lower doses of daptomycin and in particular to the range of 3 to 10 mg/kg daptomycin. Accordingly, there was no clear and unambiguous disclosure in the application as filed for the feature "once every 48 hours" in combination with the further feature of claim 1 concerning the dosage regime claimed namely the dose of "3 to 10 mg/kg of daptomycin". Therefore the main request did not meet the requirements of Art. 123(2) EPC.

2. Article 123(3) EPC

2.1 Extent of protection

2.1.1 Relationship between Article 123(3) EPC and Article 69 EPC

In T 2259/09 whilst the patent as granted comprised a set of Figures, in the amended patent according to the main request all drawings were deleted to comply with the requirements of Art. 123(2) EPC, and the description was correspondingly adapted.

Concerning the examination of Art. 123(3) EPC, with the deletion of the drawings from the patent specification, technical information was undoubtedly lost. Whilst this loss of information might also introduce some uncertainties about certain details of the preferred embodiments depicted in the drawings, it could not be concluded, as in the impugned decision, that it automatically broadened the protection conferred by the patent. In the case at issue, the claims were not limited to any of the details, dimensions or features specifically depicted in the (deleted) drawings. Moreover, under Art. 69(1) EPC, reference to the drawings of a patent specification may be helpful or even necessary if an ambiguity exists in the granted claim. However, in the case at issue, as the appellant pointed out, the impugned decision did not identify any such ambiguity; nor could the board see any. None of the specific features of claim 1 of the granted patent had a different or broader meaning without the drawings than with the drawings. Nor did deleting the drawings create any potential additional ambiguity beyond that which might have been present in claim 1 of the patent as granted. Consequently, the board came to the conclusion that the deletion of the drawings did not extend the scope of the protection conferred by the main request, which request therefore fulfilled the provisions of Art. 123(3) EPC.

In T 2284/09 the amount of acid which claim 1 of the second auxiliary request required was increased, compared to the amount of acid required by granted claim 1, by the amount needed to effect neutralisation. Consequently claim 1 of the second auxiliary request conferred protection on a process involving the addition of a different, greater amount of acid than the claim of the patent as granted. The effect of this amendment of the wording of claim 1 was, therefore, to enlarge the scope of protection conferred by the claim as compared to the patent as granted. This contravened Art. 123(3) EPC. The appellant (patent proprietor) had relied upon decision T 108/91 (OJ EPO 1994, 228), according to which, in the case of inconsistency between the claim and the totality of the disclosure, it is permissible to refer to the description and, pursuant to Art. 69(1) EPC, to rely on the disclosure of the description to amend the claim. T 108/91 was however not applicable in the case at issue, since there was no inconsistency between the claim of the granted patent and the description. Furthermore decision G 1/93 (OJ EPO 1994, 541) ruled that in the case of a non-disclosed limitation being introduced during examination proceedings (as was the case here), it is not permissible to remove it when so doing would extend the scope of protection. In G 1/93 the role of Art. 69(1) EPC was also considered and it was concluded that the description was to be used for assessing in particular sufficiency of disclosure and in determining the scope of protection conferred by the claims. However, there was no finding in G 1/93 that supported the position of the appellant that the description might be used as a repository from which amendments to the claims could be derived even if such amendments would contravene Art. 123(3) EPC. Accordingly there was no basis in the EPC or in the case law of the Enlarged Board to support the position and approach of the appellant in relying on the description in order to render permissible an amendment to the claims of the patent as granted that resulted in a change of the scope of protection. The second auxiliary request was refused.

2.1.2 Miscellaneous issues

In T 260/10, the feature "having at least one optical display unit and in particular at least one operating unit ..." in claim 1 as granted had been replaced and the words "and in particular at least one operating unit" thereby deleted. The board found that it generally depended on the specific context whether a feature following the expression "in particular" had to be regarded as optional. As a rule, an optional feature in the main claim was one which was not essential to the claimed teaching but instead served as an example illustrating other features. The wording used in claim 1 as granted meant that the claimed domestic appliance included not only an optical display unit but also an operating unit, the expression "in particular" having to be construed in the sense of "above all" or "especially". The operating unit was specified not as an example of a possible component, but rather as an essential part of the domestic appliance. In this case, "in particular" gave special emphasis to the operating unit as part of the appliance. The scope of protection was limited by this non-optional feature and its deletion therefore infringed Art. 123(3) EPC.

F. Divisional applications

The case underlying J 20/12 concerned a parent application which had been unconditionally withdrawn but the appellant had filed a request to correct this withdrawal. This request was ultimately refused by the Legal Board of Appeal (in decision J 1/11), essentially because by the time the request for correction had been filed notice of the withdrawal had already been recorded in the European Patent Register.

After the request for correction, the appellant filed a divisional application, which the Receiving Section decided not to process because the parent application had been withdrawn and, according to R. 36(1) EPC, an applicant could only file a divisional application "relating to any pending earlier European patent application." The Legal Board saw no difference between the case of an application deemed to be withdrawn due to the non-payment of fees (which had been dealt with in J 4/11, OJ EPO 2012, 516) and an application that has been voluntarily withdrawn by a communication from the applicant (as in the case at issue). In neither case was the withdrawal the result of a decision taken by the Office. From the wording of Art. 67(4) EPC ("withdrawn" in contrast to "finally refused"), and the further clarification given in decision J 4/11, it could be concluded that an application was no longer pending as from the moment it had been withdrawn. This was not altered by the possibility of filing a request for correction under R. 139 EPC, nor by the actual filing of such a request. In other words, an application was not pending because a correction of a withdrawal under R. 139 EPC had been requested. It was not necessary for the board to decide what the position would have been if the request for correction pertaining to the parent application had been allowed.

III. RULES COMMON TO ALL PROCEEDINGS BEFORE THE EPO

A. The principle of the protection of legitimate expectations

1. Obligation to draw attention to easily remediable deficiencies

In T 642/12, the opponent, a Swiss company, filed a notice of appeal in Dutch together with an English translation, and paid 80% of the full appeal fee. The formalities officer of the opposition division completed EPO Form 2701 and forwarded the case to the registry of the boards of appeal.

The board noted that the appellant as a Swiss company was not entitled to use the Dutch language and the appeal was therefore inadmissible for lack of paying the (full) appeal fee in due time. The question arose whether the timely payment of the appeal fee could be recognised on the basis of the principle of the protection of legitimate expectations.

The board concurred with the appellant to the extent that there had indeed been a potential possibility to discover the error in good time, since seven working days had been available to warn the appellant and to pay the appeal fee. However, this potential possibility did not automatically translate into a legitimate expectation on the part of the appellant. A potential possibility of discovering the error was not sufficient to establish the legitimate expectation that a registrar of the boards of appeal would warn an appellant within seven working days before the expiry of the time limit that a reduced appeal fee had been relied on in error and therefore the appeal fee was deemed not to have been paid. In addition, the fact that the appeal had been processed in the EPO, as may be inferred from EPO Form 2701, was not a basis for any legitimate expectation concerning the validity of the appeal. In case of an inter partes appeal case, completion of EPO Form 2701 by the formalities officer of the department of first instance did not establish the legitimate expectation that formal requirements of the appeal, such as the payment of the appeal fee, had already been checked by the EPO. The board concluded that timely payment of the appeal fee could not be recognised on the basis of the principle of the protection of legitimate expectations. (See also Chapter III.K.2. "Insufficient payment – small amounts lacking".)

B. Right to be heard

In T 1014/10 the appellant (patent proprietor) requested the refund of the appeal fee, arguing that during the opposition proceedings it had been given no opportunity to study the opponent's submissions because they were delivered to its office only on the day of the oral proceedings. The board observed that the EPO had sent an electronic communication to the representative of the patent proprietor to draw its attention to the new submissions.

Independently of when the notification was issued, it was the duty of the parties – and of the board – to check the content of the electronic file in order to make sure that no submission had been added in the days before the oral proceedings. Moreover, the proprietor, who received a copy of the submissions at the oral proceedings, could have requested an interruption of the oral proceedings to study their content or even asked the opposition division not to admit them into the proceedings. As shown by the minutes of the oral proceedings, the proprietor did not make use of these procedural options. Under these circumstances, the late-filed submissions, which furthermore did not contain any new facts, were simply to be put on the same footing as new arguments which might have been put forward and discussed anyway during the oral proceedings. The board did not see any violation of the patentee's rights under Art. 113 EPC.

In T 1843/11 the appellant had alleged a substantial procedural violation because a specific argument concerning sufficiency of disclosure had been raised in the written and oral opposition proceedings, but the opposition division had not addressed it at all in its decision. The board recalled that, according to R. 111(2) EPC, decisions of the EPO which are open to appeal shall be reasoned.

Although the opposition division was not required to address each and every argument of a party (see R 19/10, R 17/11), the important question was whether the party concerned could objectively understand whether the decision was justified or not. In this respect, the decision should contain at least some reasoning on crucial points of dispute, in order to give the party concerned a fair idea of why its submissions were not considered convincing and to enable it to base its grounds of appeal on relevant issues (T 70/02). The points to be addressed were, in particular, arguments which might militate against or cast doubt on the decision in question (T 246/08). The board noted that, later on in its decision, it had acknowledged sufficiency of disclosure on the basis of the appellant's argument. This argument was thus clearly crucial to the point decided upon by the opposition division. The fact that no reasons were given in the decision of the opposition division why the appellant's argument was not an answer to the objection under Art. 83 EPC therefore amounted to a substantial procedural violation. The fact that the appellant in the case at issue was given the opportunity to (and in fact did) put forward the argument was not sufficient for the requirements of Art. 113(1) EPC to be met. What was additionally important was that the opposition division demonstrably heard and considered this argument (see T 763/04); this condition had not been met in the case before the board. Thus a substantial procedural violation had occurred as regards the omission of the appellant's argument from the opposition division's decision.

For further points regarding Art. 113 EPC see Chapter IV.E.7.2.5, "Fundamental violation of Article 113 EPC (Article 112a(2)(c) EPC)".

C. Oral proceedings

1. Continuation of oral proceedings on the next day

In T 2534/10, the chairman of the opposition division had proposed, late in the evening of the only scheduled hearing date, that the oral proceedings be continued the next day.

Although the patent proprietor had objected to this proposal, the proceedings had been continued the next day, and both parties had attended. The board observed that the two months' notice to be given in summons to oral proceedings under R. 115 EPC met the parties' fundamental need to be able to plan for and estimate the likely course of those proceedings. They had the same need when it came to continuing proceedings on a calendar day other than that specified in the summons. Accordingly, a fresh summons had to be issued if proceedings were to be continued beyond the calendar day specified in the original summons.

2. Non-attendance at oral proceedings

In T 1500/10, the board held that the purpose of oral proceedings was to settle as far as possible all outstanding questions relevant to the decision and to speed up the procedure. If oral proceedings took place at the instance of the EPO because it considered this to be expedient (see Art. 116(1) EPC), the party had an obligation to attend the oral proceedings. There was no right to a solely written procedure.

Oral proceedings were themselves an opportunity for the applicant to present its comments, in accordance with Art. 113(1) EPC. If a party decided not to attend the oral proceedings without a serious reason, it chose not to make use of the opportunity to comment at the oral proceedings on any of the objections raised during them, and it had no right to make additional written submissions. Serious reasons were the same reasons that could justify postponement of oral proceedings (see Notice of the Vice-President of Directorate-General 3 of the European Patent Office dated 16 July 2007 concerning Oral Proceedings before the Boards of Appeal, Special edition No. 3 of OJ EPO 2007, 115). See also under Chapter IV.E.6. "Reimbursement of the appeal fee".

D. Re-establishment of rights

In J 5/11 the Legal Board held that the applicant for re-establishment of rights who failed to substantiate his request adequately in the first-instance proceedings, in particular after being expressly invited to do so, could not normally make good that failure by submitting additional evidence with the grounds of appeal. While it was true that the primary function of the boards was to give a judicial decision on the correctness of a first-instance decision of the Office (see J 18/98), that did not necessarily mean that new evidence submitted for the first time on appeal was automatically inadmissible. A rigid rule excluding all new evidence on appeal might lead to injustice and unfairness in some cases and would not be compatible with the principles of procedural law generally recognised in the Contracting States (cf. Art. 125 EPC).

E. Language regime

1. Derogation from the language of the proceedings in oral proceedings – provision of simultaneous interpretation

In appeal T 2422/10 (language of proceedings English), the representatives of both parties were German native speakers and all the written submissions had been in German. The appellant's (opponent's) representative announced that he would use German at the oral proceedings. The respondent's (proprietor's) representative announced that he would speak English and would be accompanied by an expert, the latter to address the board, and also requested simultaneous interpretation from German into English. However, the board did not allow the request for either the representative or the accompanying person.

The respondent had argued that it had an absolute right to interpreting into the language of the proceedings according to R. 4(5) EPC if the other party employed a different language. The expression "if necessary" in R. 4(5) EPC was to be interpreted in combination with R. 4(1) EPC in this sense, and the board had no discretion. The board disagreed. The general rule needed to be set against the principle of efficiency of the proceedings and the duty of all services of the EPO, including the boards of appeal, to observe the finances of the EPO. The board considered the wording of R. 4(5) EPC to be such that it allowed the board to assess the necessity of such interpretation (see T 131/07). It was evident that the respondent's representative was quite capable of understanding any oral submissions made by the appellant in German without the need for interpretation.

The board decided not to hear the accompanying person following the criteria of G 4/95 (OJ EPO 1996, 412) since the person was intended to speak on issues reserved for the professional representative. The arrangement of interpretation for an accompanying person at the expense of the EPO would not be justified because, in the board's opinion, accompanying persons did not have an automatic right to interpretation. This could for instance depend on whether the board intended to let them address the board (see T 131/07). Providing interpreting merely to suit the convenience of a party was not a sufficient reason (see T 418/07).

2. Language privilege – fee reduction

See T 642/12, reported in chapter III.K. "Rules relating to fees".

See also the amendments to R. 6 EPC introduced by decision of the Administrative Council (CA/D 19/13) of 13 December 2013 (OJ EPO 2014, A4).

F. Law of evidence

1. Witness testimony and expert opinion – Time frame for submitting evidence

In the oral proceedings before the board in T 480/11, the appellant (proprietor) had requested that a Mr J. be allowed to speak about "their experience of repeating the claimed invention" in relation to claim 1 of a newly presented auxiliary request. The board stated that it was aware that the appellant had referred to Mr J. as a technical expert in its letter of four weeks before the oral proceedings and had requested that he be allowed to comment on any technical issue relating to the claimed invention that might come up during the hearing. However, the subject on which he intended to speak according to the appellant's request as specified during the oral proceedings was not simply a technical issue but an event in the past, namely the performance of experiments and the results obtained thereby, which had taken place at the appellant's laboratory. So the appellant's request was actually directed to hearing Mr J. as a witness rather than as a technical expert.

The appellant argued that the proprietor had made a statement during the opposition proceedings similar to the one which Mr J. was going to present in the appeal. However, the statement had been challenged by the respondent in its response to the statement of grounds of appeal. So, at the latest upon receipt of this letter, the appellant must have been aware that its statement during the opposition proceedings might not be sufficient as proof and that thus more evidence would be required. Nevertheless, it chose to wait until the latest possible point in time during the appeal proceedings to offer further evidence in the form of a "witness statement". If the witness had been allowed to speak, the respondent would have had to be given sufficient opportunity to challenge the witness statement and it would have been necessary to adjourn the oral proceedings. Therefore, the board decided not to allow the appellant's request to hear Mr J., pursuant to Art. 13(3) RPBA.

G. Suspected partiality

1. Suspected partiality of members of the opposition division

The general principle that nobody may decide a case in which a party may have good reasons to suspect him of partiality applies not only to the members of the boards of appeal under Art. 24(1) EPC but also to the members of the EPO's departments of first instance involved in taking decisions affecting the rights of any party (cf. G 5/91, OJ EPO 1992, 617).

In T 1674/12, opponents 1 and 3 alleged a series of substantial procedural violations, in particular partiality of the opposition division. They inferred partiality from a number of circumstances, one being that a member of the division, namely the primary examiner (i.e. not the chairman), had been involved in granting a divisional patent application resulting from the opposed patent. The opponents maintained that, since this member had granted the divisional application without awaiting the decision on that earlier patent, even though he had been aware of the objections to it, he at least had been biased.

The board observed, and the opponents conceded, that, whilst the rules applicable to board members (Art. 24(1) EPC) were different, a member of an examining division which had granted a patent could take part in opposition proceedings concerning the same patent, provided he did not act as chairman (see Art. 19(2) EPC). Such participation was therefore not a sufficient basis for objecting to that member or suspecting him of partiality.

The opponents based their allegation that the primary examiner had been biased on the contention that he could not decide on a divisional application before knowing the outcome of the opposition to the earlier patent from which it resulted. Rather, he ought to have taken that outcome into account in deciding on the divisional application. The board observed that the decision to grant a patent was taken by the examining division and not the primary examiner alone. In its view, requiring that the division always await the outcome of a different, albeit related, case would run counter to its independence. Clearly, therefore, the examiner's decision to process the divisional application could not be regarded as a sure sign of partiality.

At the oral proceedings, the opponents claimed that, even if none of the various circumstances was enough on its own to establish partiality of the opposition division, as a whole they showed that the proprietor had been systematically favoured. The board, however, observed that, according to case law, partiality could not be established solely on the basis of a party's subjective impression, and that the circumstances also had to be considered from an objective observer's point of view (see T 190/03, OJ EPO 2006, 502 and T 281/03, both of 18 March 2005).

2. Suspected partiality of members of the boards of appeal

In T 1677/11 partiality objections were raised against all board members under Art. 24(3) EPC. It was argued that, since the substantively identical parallel appeal case T 1760/11 had been decided by a board in identical composition one week previously, the board would not bring an open mind to the appeal now before it.

The board first had to decide whether the said objections were admissible. It noted that according to Art. 24(3) EPC, second sentence, "an objection shall not be admissible if, while being aware of a reason for objection, the party has taken a procedural step". The principle underlying this provision is that an objection should be "raised immediately after the party concerned has become aware of the reason for the objection", since "otherwise, the system could be open to abuse" (see G 5/91, OJ EPO 1992, 617).

The board noted that the respondents were aware of the closely related parallel appeal T 1760/11 right from the beginning of the current appeal proceedings. The composition of the boards was identical in both cases, as provided in Art. 7 of the business distribution scheme (see Supplement to OJ EPO 1/2012). However, no concerns had been voiced by any of the parties in this respect. It was only after an adverse decision in that case had been announced that the respondents raised their objections of suspected partiality in the case at issue. The board also stated that regardless of whether the respondents had taken a specific procedural step in the current appeal proceedings, it was undoubtedly the case that they had not submitted their objection immediately after becoming aware of the reasons. It held that, in view of the fact that the objections raised were linked to both appeals, attendance at oral proceedings in T 1760/11 had to be regarded, in the factual context of the case now at issue, as a procedural step within the meaning of Art. 24(3) EPC. Thus, in view of their timing, the objections under Art. 24(3) EPC were rejected as inadmissible.

3. Suspected partiality of members of the Enlarged Board of Appeal

Art. 24 EPC applies to both members of the boards of appeal and members of the Enlarged Board of Appeal. The latter are therefore covered both in their capacity under Art. 112 EPC and in the additional capacity later conferred on them under Art. 112a EPC.

In R 2/12 the objection of suspected partiality was derived exclusively from the text of the communication drafted by the rapporteur by which the petitioner was informed of the Enlarged Board's provisional opinion on the petition. A suspicion of partiality must be justified on an objective basis (following G 2/08 of 15 June 2009, unpublished), i.e. a reasonable onlooker considering the circumstances of the case would have to conclude that the party might have good reasons to doubt the impartiality of the member objected to (following G 1/05, OJ EPO 2007, 362). The Enlarged Board found there was nothing in the communication which could justify any suspicion of partiality. It would be incompatible with an objective assessment of a case and with the principle of a fair trial in inter partes proceedings, if a board member could be "deposed" on the ground that they did not opine in favour of a particular party right from the beginning of the proceedings.

H. Formal aspects of decisions of EPO departments

1. Completion of the internal decision-making process

In G 12/91 (OJ EPO 1994, 285), the Enlarged Board of Appeal had decided that the decision-making process following written proceedings is completed on the date the decision to be notified is handed over to the EPO postal service by the decision-taking department's formalities section. In T 2573/11 the appellant submitted that, following G 12/91, as well as T 556/95 (OJ EPO 1997, 205) and T 394/96 and established EPO practice, the date of termination of proceedings was three days prior to the date of actual despatch of the decision which was stamped at the upper right-hand side of the decision, irrespective of the date appearing in the box marked "to EPO postal service".

The board disagreed – if it was clearly indicated in the decision on which date the formalities section handed the decision over to the EPO postal service, this date was directly brought to the knowledge of the parties and was thus the date on which written proceedings before the decision-making department were completed. This conclusion was in line with G 12/91; in the case underlying G 12/91 the date on which the formalities section had handed over the decision to the EPO postal service had not been indicated.

I. Correction of errors in decisions

G 1/10 was concerned with the procedural issue of whether the patent proprietor's request for correction of the grant decision under R. 140 EPC, which was filed after the initiation of opposition proceedings, was an admissible remedy on which only the examining division has the power to take a binding decision. The Enlarged Board of Appeal answered that, since R. 140 EPC is not available to correct the text of a patent, a patent proprietor's request for such a correction is inadmissible whenever made, including after the initiation of opposition proceedings. The Enlarged Board's decision is limited to corrections within grant decisions of the description, claims and drawings (patent documents), but is not concerned with corrections to bibliographic data. Therefore, the Enlarged Board concluded that R. 140 EPC is not available for correcting patents, including during opposition or limitation proceedings. However, it is always open to a patent proprietor to seek to amend his patent during opposition or limitation proceedings and such an amendment could remove a perceived error. Such an amendment would have to satisfy all the legal requirements for amendments including those of Art. 123 EPC.

J. Inspection of files and stay of proceedings

1. Inspection of files

1.1 Exclusion from file inspection

In T 1839/11 the situation was unusual and, so far as the board was aware, unique in proceedings before the Office, in that the respondent had obtained the documents subject to the request from the appellant as part of the English disclosure system but subject to an obligation of confidence and before it was known whether the respondent would be released from this obligation after a trial in the English proceedings. Both parties had been ordered by the English court to use their best endeavours to keep the disclosure documents confidential to the extent that inter alia the EPC allows.

Nevertheless, the respondent was not prevented from making use of the documents in these appeal proceedings, and indeed was expressly allowed to do so. The documents which were sought to be excluded from public file inspection contained in part information which did not serve the purpose of informing the public about the patent and the board also stated that their publication would be likely to prejudice the economic interests of the appellant and the respondent respectively.

The board came to the conclusion that where a filed document contains information, some of which serves the purpose of informing the public about the patent but some of which does not, the filing of a version of the document in a form from which the latter information has been redacted may form the proper basis for an order excluding the unreacted document from file inspection under R. 144 EPC, the redacted version being open to file inspection.

In T 99/09 the document in question contained precise technical details, particularly with regard to the source of the constituents of a marketed medicinal product and certain manufacturing specifications relating to it. Given the technical nature of the document the board concluded that its disclosure might indeed be prejudicial to the economic interests of the appellant.

In other respects, the request to exclude this document from inspection was neither objected to, nor commented on, by the respondents. The relevant document was therefore excluded from inspection in the light of Art. 128(4) and R. 144(d) EPC, in accordance with Art. 1(2)(a) of the Decision of the President of the EPO dated 12 July 2007 (Special edition No. 3 of OJ EPO 2007, J.3).

2. Stay of proceedings under Rule 14(1) EPC

2.1 General

In J 9/12 the board decided that it is not possible for a person seeking a decision within the meaning of Art. 61 EPC to apply to the EPO under R. 14(1) EPC for an automatic and immediate suspension of proceedings in relation to any divisional application deriving from an earlier application by way of an extension of the stay of the proceedings in relation to the earlier application.

The EPO is entitled to stay the proceedings for grant under R. 14(1) EPC only if there is clear and unambiguous proof that the claimant's request in the proceedings before the national court is that he is entitled to the grant of the European patent application that is to be suspended, and not for any other application, however related it may be. Moreover, only those proceedings for grant may be stayed that concern European patent applications pending at the date of filing of the request for stay.

Furthermore, proceedings for grant cannot be stayed before the publication of the European patent application. Therefore, R. 14(1) EPC cannot be regarded as conferring on the departments of the EPO the power to extend ex officio a stay of the grant proceedings concerning an earlier application to any divisional application proceedings filed thereafter.

The case law of the Legal Board of Appeal is consistent in that a request for a stay of the proceedings under R. 14(1) EPC may be filed up to the day before the date of the mention of grant of the European patent in the European Patent Bulletin (J 7/96, OJ EPO 1999, 443; J 36/97). A stay of proceedings under R. 14(1) EPC takes immediate effect as from the date on which an allowable request is filed. The EPO is responsible for the grant proceedings up to the date of the publication of the mention of grant of the patent. This competence includes the publication of any necessary correction of a publication of the mention of grant (J 15/06). As a consequence, the publication of the mention of grant does not take away the competence of the Legal Division to issue a communication and, if requested, a decision ordering a stay of the grant proceedings, provided that an allowable request is filed before publication (J 15/06, J 33/95, J 36/97). For the above reasons, the board did not accept the argument that a stay of proceedings for grant communicated after the publication of the mention of grant is to be regarded as automatically null and void. The appeal was dismissed.

2.2 Rule 14(3) EPC

Referring to decision J 33/03, the board in J 13/12 noted that, in contrast to the position under R. 14(1) EPC, R. 14(3) EPC gives the EPO the discretion to decide whether proceedings are to be resumed.

In exercising its discretion under R. 14(3) EPC, the EPO, i.e. the Legal Division and, under Art. 111(1), second sentence, EPC, also the board, in exercising powers within the competence of the Legal Division, is obliged to weigh up the interests of the applicant, on the one hand, against those of the party who brought national entitlement proceedings against the applicant, on the other. In doing so, the EPO must observe the Art. 61 EPC objective of a "well-balanced and fair resolution of conflicting interests" (G 3/92, OJ EPO 1994, 607). The Guidelines for Examination can also be used as internal administrative instructions.

The Legal Division thus did not act improperly in ordering the resumption of proceedings for grant without first setting a date for the said resumption in a separate communication; rather it acted in accordance with the EPO's internal regulations (Guidelines for Examination) governing the exercise of discretion. The first time the Legal Division was able to set a date was in the impugned decision.

Nor can it be derived from the Guidelines for Examination that before setting a date for the resumption of proceedings for grant, or at any rate before actually resuming proceedings, the Legal Division should have waited for the definitive closure of entitlement proceedings, where applicable after the exhaustion of all legal remedies (first-instance proceedings, appeal proceedings), or that it should have at least waited for the closure of appeal proceedings.

K. Observations by third parties

1. Anonymously filed observations in appeal proceedings

In T 1439/09 anonymous, unsigned third-party observations were filed during the appeal proceedings.

In decision T 146/07, this board, in a different composition, had held that the identification of a third party in the context of third-party submissions in opposition proceedings was particularly important in order to allow the competent organ of the EPO to verify whether the observations were indeed filed by a third party rather than by a party to the proceedings.

Otherwise, a party might be tempted to submit late observations and/or documents by means of anonymous third-party observations in order to avoid negative procedural consequences such as apportionment of costs. Moreover, unsigned submissions by a party to the proceedings were deemed not to have been filed if, after a communication according to R. 50(3) EPC had been sent out by the EPO, they were not signed in due time. Since unsigned anonymous third-party observations did not allow the EPO to send out such an invitation, they necessarily remained unsigned. As a consequence, they were deemed not to have been filed. In the case at issue, the board saw no need to depart from this line of reasoning. Therefore, the anonymous observations filed under Art. 115 EPC were deemed not to have been filed and were disregarded by the board.

2. Third-party observations filed at a late procedural stage

In T 637/09 the board stated that a third party within the meaning of Art. 115 EPC is not a party to the proceedings. Therefore, the admission into the appeal proceedings of third-party observations filed in the course of these proceedings is at the board's discretion. When exercising its discretion the board took into account that it should not accord the third party within the meaning of Art. 115 EPC more favourable treatment than would be given to an actual party seeking to introduce such submissions at that stage of the proceedings.

The third-party observations filed less than one month before the oral proceedings, as well as the third-party observations filed one month before the continuation of the oral proceedings, were not admitted into the proceedings since their admission would have accorded the third party more favourable treatment than would have been given to an actual party. There was no justification for such a late filing. These observations contained inter alia new submissions which would, if admitted at that late stage of the proceedings, have compromised the fairness of the proceedings.

3. Third-party observations inadmissible in review proceedings

In the proceedings for petition for review R 18/11 third-party observations were filed. The chairman of the Enlarged Board declared at the beginning of the oral proceedings that the Enlarged Board considered these observations to be inadmissible, because according to Art. 115 EPC such submissions had to concern patentability, and patentability issues could not be the subject of review proceedings.

L. Rules relating to Fees

1. Methods of payment – debit orders

J 14/12 concerned a request for reimbursement of the additional fees in respect of annual renewal fees in a European divisional application (cf. R. 51(3) EPC). These had been paid from a deposit account held with the EPO.

In a letter filed within the four-month period of R 51(3), second sentence, EPC, the representative had only requested debiting of "the fees which fell due for payment with the filing of the above-mentioned divisional application" from a numbered deposit account.

Taking the letter in conjunction with the "Internal fee calculation sheet" submitted at the time the application was filed, the board found, as regards the renewal fees for the 3rd to 10th years, that the conditions for a valid debit order were fulfilled (see in particular point 6.3 of the Arrangements for deposit accounts (ADA) and their annexes (valid as from 1 April 2009), Supplement to OJ EPO 3/2009). Consequently, the date to be considered as the date on which payment for those renewal fees was made was within the four-month period, and no additional fee fell due.

However, the same did not apply to the renewal fees for the 11th and 12th years, as the purpose and amount of each of these fees were given in neither of the above documents. No different conclusion could be drawn having regard to decisions T 170/83 (OJ EPO 1984, 605) or T 152/82 (OJ EPO 1984, 301).

Note: According to the Arrangements for deposit accounts (ADA) in the version valid as from 1 April 2014, the use of the relevant standard form (EPO Form 1010 or PCT Form PCT/RO/101 or IPEA/401) is now mandatory for debit orders filed on paper or by fax (see points 6.2 and 6.3 ADA and the Notice dated 11 February 2014, supplementary publication 4, OJ EPO 2014).

2. Insufficient payments – small amounts lacking

In case T 642/12 the appellant, registered as a company having an address in Switzerland, had filed the notice of appeal in Dutch and paid a reduced appeal fee pursuant to R. 6(3) EPC. It subsequently did not dispute that it was not entitled to benefit from the fee reduction. Instead, it contended, inter alia, that the EPO must accept the partial payment and overlook the missing 20% through the application of Art. 8, last sentence, RFees.

The board disagreed, its conclusion being summarised in point 3 of the catchword to the decision as follows:

"Small amounts lacking" in Art. 8 RFees are to be read as "insignificant or negligible" amounts. The legislator presumed that a fee reduction of 20% pursuant to R. 6(3) EPC is not merely a symbolic one, but will effectively alleviate the burden of having to prepare translations. Therefore the legislator could not have intended this fee reduction to be considered small in the sense of negligible or insignificant. See also T 905/90 (OJ EPO 1994, 306).

Note: R. 6(3) EPC and Art. 14(1) RFees have been amended by decision of the Administrative Council (CA/D 19/13) of 13 December 2013 (entry into force: 1 April 2014), OJ EPO 2014, A4.

3. Partial refund of the examination fee

Reference is made to Chapter IV.B. for decision J 9/10, in which the Legal Board of Appeal dealt with the question of when "substantive examination" begins for the purpose of a partial refund of the examination fee under Art. 10b(b) RFees (cf. now Art. 11(b) RFees) – see also earlier decision J 25/10 (OJ EPO 2011, 624).

Attention is also drawn to the notice from the EPO dated 29 January 2013 concerning adjustments to the system for search and examination fee refunds (Art. 9(1) RFees and Art. 11(b) RFees) following decisions J 25/10 and J 9/10 of the Legal Board of Appeal (OJ EPO 2013, 153).

M. Representation

1. Professional representatives

1.1 Duty of persons without residence or place of business within a contracting state to be represented by a professional representative

In J 9/13, the Receiving Section refused the European patent application pursuant to Art. 90(5) EPC, because the applicant, resident in Moscow, had not appointed a professional representative as required by Art. 133(2) EPC.

The appellant (applicant) contended that Art. 133(2) EPC did not apply to residents of the Russian Federation. The Russian Federation was a party to the "Partnership and Cooperation Agreement (PCA) between the EU countries and the Russian Federation" and the WTO/TRIPS Agreement. In view of these international agreements, as a resident of the Russian Federation the applicant was entitled to treatment no less favourable than that enjoyed by EU countries' nationals with regard to patenting under the EPC.

The board observed that both the notice of appeal and the statement setting out the grounds of appeal were co-signed by Mr. D. P., who appeared to have drafted these texts, but who had not been shown to be an authorised representative, and by Mr. N. N. himself, who was the applicant and appellant in this case. Under these circumstances the existence of a valid appeal could only be recognised if the appellant's contention that he was not obliged under Art. 133(2) and 134 EPC to be represented by a professional representative in proceedings under the EPC was correct.

The board stressed that neither the European Patent Organisation nor the European Patent Office is part or member of the European Union. This was inter alia confirmed by the Enlarged Board of Appeal in decision R 1/10. Therefore, neither the European Patent Organisation nor the European Patent Office are bound by the provisions of the PCA, neither of them is a "party" or "other party" pursuant to Art. 98 of the PCA. For this reason alone, this international agreement could not serve as a legal basis for according to the appellant the same treatment as that which is accorded to residents of EPC countries with regard to proceedings established by the EPC. The corresponding reasoning applied with respect to the WTO/TRIPS Agreement. General multilateral treaties such as the TRIPS Agreement are a source of international law for the contracting parties and for no one else. Furthermore the board said that in legal terms neither the legislation of the contracting states nor the international conventions signed by them are part of the autonomous EPC system.

Thus the appellant was obliged to be represented by a professional representative pursuant to Art. 133(2), 134 EPC in proceedings established by the EPC. Since Art. 133(2) EPC is also a mandatory provision in appeal proceedings and since no appointment of a professional representative was notified to the European Patent Office within the three-month time limit set in the Registrar's communication, the current appeal was deemed not to have been filed (R. 152(6) EPC by analogy).

2. Authorisations for appointment of a representative

In T 637/09, the appellant informed the board and the respondent that Mr Sc., a professional representative, would represent the appellant jointly with Mr M., the professional representative already acting on behalf of the appellant. Mr Sc.'s entitlement to act as a representative for the appellant was contested by the respondent at the beginning of the oral proceedings. The board observed that, as can be inferred from R. 152(10) EPC, a party may be jointly represented by several representatives. Further, the appointment of Mr Sc. as joint representative was not a change of representative within the meaning of Art. 1(2) of the Decision of the President of the European Patent Office dated 12 July 2007 on the filing of authorisations (Special edition No. 4 of OJ EPO 2007, 128). Thus, in order to represent the appellant, Mr Sc was not required to file a signed authorisation on the basis of Art. 1(2) of the decision. The board also had no reason to doubt Mr Sc.'s entitlement to act for the appellant. The circumstances of the particular case thus did not necessitate the production of a (further) authorisation in accordance with Art. 1(3) of the decision. The board concluded that Mr Sc. was duly authorised to represent the appellant in addition to Mr M.

2.1 Filing of authorisation

If a European representative files an opposition on behalf of a party but fails to file a signed authorisation in due time in response to a request to do so from the board, the opposition is deemed not to have been filed (R. 152(1) and (6) EPC).

The board in T 1700/11 ruled accordingly, after setting out of very detailed reasons. It also found that this legal fiction resulted in a loss of rights (R. 112(1) EPC) and that an examination of the admissibility of the opposition, which had not been filed according to the legal fiction, was out of the question.

In T 1542/10, a letter was filed on 10 January 2011 by professional representative P. The letter indicated "Nokia Siemens Networks OY" as opponent and used the same internal reference number as that indicated by representative B in the notice of opposition. In addition it referred to an enclosed power of attorney. According to this authorisation representative B authorized P to represent "Nokia Siemens Networks GmbH & Co. KG" as opponent before the EPO in respect of the said European patent. In reply, the appellant (patent proprietor) took the view that the letter which had been filed by representative P on 10 January 2011 did not qualify as a written reply by the respondent for the purposes of Art. 12(1)(b) RPBA and should instead be regarded as a third party observation pursuant to Art. 115 EPC. It was argued that the letter was submitted on behalf of a legal person different from the respondent since the enclosed authorisation did not indicate the respondent's name but that of Nokia Siemens Networks GmbH & Co. KG.

The appellant further maintained that the respondent having thus failed to reply to the grounds of appeal had lost its status as a party as of right according to Art. 107, second sentence, EPC or at least its entitlement to submit observations regarding the appellant's case set out in the statement of grounds of appeal. Under the given circumstances it was quite obvious to the board from an objective perspective that the indication of the name Nokia Siemens Networks GmbH & Co. KG in the authorisation was made erroneously. The fact that the board's registry did not receive an answer after it had sent out a communication pointing out that no valid authorisation had been submitted by the respondent's new representative did not entail any negative consequences for the respondent. In particular it was noted that the communication did not specify any time limit so that the sanction provided for by R. 152(6) EPC could not apply.

Neither was the lack of response a sufficient reason for retrospectively giving a different meaning to the letter dated 10 January 2011 since that meaning had to be assessed objectively as understood at the time when the letter was received. In view of the authorisations submitted by the respondent in its letter dated 13 June 2013 and at the oral proceedings, the board had no doubt that representative P had been authorised to represent the respondent. It followed that the letter dated 10 January 2011 had to be regarded as a written reply to the appellant's statement of grounds of appeal for the purposes of Art. 12(1)(b) RPBA.

IV. PROCEEDINGS BEFORE THE EPO

A. Preliminary and formalities examination

1. Accordance of a date of filing

1.1 Subsequent filing of missing parts of the description or missing drawings (Rule 56 EPC)

1.1.1 Interpretation of Rule 56 EPC

According to J 27/10, the same interpretation is to be given to the term "parts of the description ... appear to be missing" in R. 56(1), first sentence, EPC as to the term "missing parts of the description" in the subsequent paragraphs of R. 56 EPC for deciding if a part is missing from the description. The term "description" in "missing parts of the description" in R. 56 EPC refers to the description which was originally filed in order to obtain a filing date and not to any other description. The incomplete originally filed description is to be completed by the missing parts, which must be added to the already filed text of the description. Thus, an interpretation of R. 56 EPC that some, or all, of the description that was originally filed in order to obtain a filing date could be amended, replaced or deleted is incorrect.

In J 15/12 the Receiving Section correctly stated that R. 56 EPC does not allow any replacement of drawings; to interpret the rule such that some, or all, of the application documents that were originally filed in order to obtain a filing date could be amended, replaced or deleted would be incorrect (see J 27/10). R. 56(1), first sentence, EPC refers to drawings referred to in the description or in the claims, which appear to be missing. Hence, where a drawing referred to in the description is missing in the application documents as filed, R. 56 EPC is applicable and the said drawing can be filed later in accordance with the procedure the rule lays down.

However, in J 2/12 the Legal Board found that where (a) the description as filed with an application includes references to numbered drawings and (b) drawings with corresponding numbering are also filed with the application, different drawings may nevertheless be filed later under R. 56 EPC as "missing drawings" if it can be established without having to apply technical knowledge that the drawings originally filed with the application are not the drawings referred to in the description and that the later-filed drawings are the drawings referred to in the description. In the case at issue, it was immediately apparent that the figures referred to in the description were not the figures as originally filed. In other cases it might be that such a conclusion could not be reached so readily, but as it was so clear in this case, the Legal Board did not have to consider where in general the dividing line should be drawn.

B. Examination procedure

1. The first stage of substantive examination

1.1 Beginning of "substantive examination"

In J 9/10 the board held that a communication pursuant to Art. 94(3) EPC on EPO Form 2001A, which was automatically generated by a computer and posted by a formalities officer without the involvement of an examiner appointed to the examining division, did not constitute a legally effective act of the examining division and therefore could not be regarded as the beginning of "substantive examination" pursuant to Art. 10b(b) RFees (inserted by decision of the Administrative Council of 10 June 1988, as last amended by decision of the Administrative Council of 15 December 2005).

The fulfilment of the second condition for the refund of the examination fee according to Art. 10b(b) RFees, gave rise to the questions of what "substantive examination" was and what kind of act or acts amounted to the beginning of "substantive examination". The board noted that to ensure predictability and verifiability of the application of Art. 10b(b) RFees, the beginning of "substantive examination" had to be interpreted as requiring a concrete and verifiable act of the examining division as regards "substantive examination" after having assumed responsibility for the examination of the application (J 25/10). The board held that if a communication of a particular examining division was to be legally valid, it had to have been written on behalf of and represent the views of the members who had been appointed to that division to examine the issues forming the subject of the communication. There was, however, no indication in the file that the appointed primary examiner had actually authenticated the communication under Art. 94(3) EPC before it was despatched by the formalities officer. Therefore, the communication could not be attributed to the examining division, but only to the formalities officer whose name was indicated on EPO Form 2001A.

1.2 Amendments under Rule 137(3) EPC

1.2.1 Discretion of the examining division under Rule 137(3) EPC

Under Art. 123(1) EPC, a European patent application or a European patent may be amended in proceedings before the EPO, in accordance with the Implementing Regulations. R. 137(3) EPC is of particular relevance.

In T 937/09, the board held that, under the second sentence of R. 137(3) EPC (as adopted by decision of the Administrative Council of 7 December 2006; see 13th edition of the European Patent Convention, European Patent Office, 2007), it was for the examining division to decide in a proper exercise of discretion whether to admit amendments to a European patent application after the first communication. In the board's view, this meant that the examining division had to admit amendments filed by the applicant together with the reply to a communication giving reasoned notice of a particular deficiency for the first time, if it could already have raised an objection to that deficiency in its first communication, and if the amendments could be regarded as an objectively suitable attempt to correct the deficiency.

In T 996/12 the appellant objected that the examining division exercised its discretion under R. 137(3) EPC not only unreasonably, but also applying the wrong principles as to the inadmissibility of the last amendments.

The board stated that, in general terms, the way in which the examining division should exercise its discretion to allow an amendment of an application depends upon the circumstances of each individual case and on the stage of the pre-grant procedure which the application has reached.

It noted that a board of appeal reviews this exercise of discretion not in the form of how it would have decided in the case, but whether it was exercised according to the wrong principles, without taking into account the right principles or whether it was exercised in an unreasonable way. In this respect, the board noted that the applicable Guidelines did not support the concept of converging claim sets being a requirement for allowing amendments or conversely, a diverging claim set not being acceptable.

The examining division referred to two board of appeal decisions, T 1685/07 and T 745/03, for this criterion. Apart from the fact that these decisions apply particularly to the situation where a patent proprietor submits a plurality of auxiliary requests in addition to a main request in opposition appeal proceedings, it needed to be stressed that its underlining justification was to be found in the efficiency of these appeal proceedings, the latter being of a judicial nature in contrast to the purely administrative character of the proceedings before, for instance, the examining division or the opposition division. The board found that this fundamental distinction eo ipso made the unqualified application of this criterion by the so-called first instance of the EPO questionable. The appellant's objection that the examining division had exercised its discretion wrongly as to the inadmissibility of the last amendments was held not well-founded.

In T 573/12 the board noted that according to R. 137(3) EPC (in the applicable version of December 2007, which is identical to former R. 86(3) EPC 1973) any amendment after the reply to the first communication needs the consent of the examining division.

This means that the examining division has discretion. One of the criteria applied in this context and accepted by the boards of appeal is whether the amendment prima facie overcomes objections previously raised. However, as set out in G 7/93 (OJ EPO 1994, 775), an examining division, when considering whether to admit an amendment or not, is required to consider all relevant factors which arise in the case. In particular it must consider and balance the applicant's interest in obtaining a patent and the EPO's interest in bringing the examination procedure to a close.

In the case at issue, the examining division's first communication was rather vague and assessed the invention in rather general terms, without analysing the individual features. The appellant's subsequent amendment added some features to the independent claims. In the board's judgment, this was a bona fide reaction. Additionally, it was the first amendment which was dependent on the examining division's consent and the minutes showed that the examining division could and did discuss the added features. Admitting the request would not have involved excessive or unjustified additional work. Furthermore, the Guidelines for Examination (Part C-VI, 4.7 – April 2010 version) stated that, "Regarding less extensive amendments, the examiner should adopt a reasonable approach, trying to balance fairness to the applicant against the need to avoid unnecessary delay and excessive and unjustified additional work for the EPO." Given the above circumstances, the board held that the examining division did not properly balance all relevant factors and thus could not endorse the examining division's conduct.

In T 158/12 the appellant's view was that there was no article or rule in the EPC which would prevent the applicant from changing from one invention to another – if they were searched – during examination.

The board considered, however, that the articles and rules present in the EPC form a statutory system which clearly leads to the conclusion that there is no provision allowing the payment of multiple examination fees for a patent application. The board stated that only one examination is to be carried out for one application, as only one single examination fee has been paid. Having once chosen one invention (or one group of inventions) to be made the subject of examination, this choice cannot be altered once examination of that invention has commenced. The board was of the opinion that this approach based on the statutory provisions of the EPC was confirmed in opinion G 2/92 (OJ EPO 1993, 591, point 2 of the Reasons). Hence, the appellant's view that the examination of an application could be based on more than one invention was not supported by this opinion of the Enlarged Board of Appeal.

2. The grant stage in examination

2.1 Communication under Rule 71(3) EPC: the text for approval

In T 1849/12 the appellant asked for the withdrawal of the examining division's communication, and requested that the examining division be instructed to issue the communication under R. 71(3) EPC without delay – specifically before the expiry of 18 months from the priority date – and therefore to grant the European patent as quickly as possible.

The board noted that Art. 93(2) EPC provided for the possibility of granting a patent before the expiry of the 18-month period. An earlier grant was thus not ruled out, provided the examining division had already concluded that the application met all the requirements of the EPC.

As this was not yet so in the case at issue, the possibility provided for in Art. 93(2) EPC of granting a patent before the 18 months expired did not apply.

The board did not see any conflict in this regard between the provisions of Art. 93(2) and 97(1) EPC. Under the Convention, the granting of a patent was conditional upon the mandatory examination of all EPC requirements. A patent could only be granted if the examining division considered all examination requirements met.

Contrary to the appellant's claim, the examining board had no discretion in this regard. It had to be taken into account that the EPO was responsible for safeguarding the interests of the public, in addition to those of the appellant, and the public had to be able to rely on it doing so. Nor was the requirement of novelty over the prior art according to Art. 54(3) EPC a subordinate one, as the appellant appeared to claim. Rather, the lack of novelty over prior art according to Art. 54(3) EPC was a ground for both opposition under Art. 100(a) EPC and revocation under Art. 138(1) EPC.

2.2 Correction of the withdrawal of the application under Rule 139 EPC

In J 6/13 it was undisputed that the applicant's letter was an unambiguous withdrawal of the application. The board pointed out that an applicant is bound by its procedural acts notified to the EPO provided that the procedural statement was unambiguous and unconditional (J 19/03).

The board held that there could be no retraction of a withdrawal if there was no reason for third parties to assume that the withdrawal was erroneous. It referred to decision J 12/03 (citing with approval decision J 25/03, OJ EPO 2006, 395), where it was stated that "a request for retraction of a letter of withdrawal of a patent application is no longer possible if the withdrawal has been mentioned in the European Register of Patents at the time the retraction is applied for if, in the circumstances of the case, even after a file inspection there would not have been any reason for a third party to suspect, at the time of the official notification to the public, that the withdrawal could be erroneous and later retracted".

The board pointed out that in the interest of legal certainty for third parties, and taking into account the public function of the register, a third party upon file inspection must have had good reason to suspect that the withdrawal was made in error in order to allow a retraction thereof.

Accordingly, the board had to determine whether, in the current case, such good reason was present. The board took the view that, based on decisions J 12/03 and J 18/10, the prospects of the application, however promising, were insufficient to infer an obvious or even potential contradiction with a subsequent withdrawal. Patent applications may be withdrawn due to considerations of business strategy, investor preference, shift in portfolios, agreements with competitors, etc. Due to financial considerations, most granted European patents are validated only in a limited number of countries. These considerations may come into play at any time, even after the recent payment of annuities, or after the communication of a positive search report. The favourable prospects of the application in this case would thus not lead a third party to the conclusion that the withdrawal was possibly made in error. Nor did they lead the representative who handled the case and who communicated the withdrawal to this conclusion, either. Therefore, the appeal was dismissed.

2.3 Claims fees due in response to Rule 71(3) EPC communication

According to R. 71(4) EPC if the European patent application in the text intended for grant comprises more than fifteen claims, the examining division shall invite the applicant to pay claims fees in respect of the sixteenth and each subsequent claim within the period under paragraph 3 unless the said fees have already been paid under R. 45 or 162 EPC.

The appeal in J 6/12 was lodged against the decision by the formalities officer acting for the examining division to refuse a request for a refund under R. 71(5) EPC of ten claims fees following amendments which had resulted in a smaller number of claims. The appellant did not dispute that R. 71(6) EPC, which, in this case, applied as worded in the revised EPC which had entered into force in December 2007, was the legal basis for the invitation to pay claims fees. According to this wording, the "text [of the application] intended for grant" was the basis for charging additional claims fees. However, the Legal Board held that this text had only become final after the examining division had approved the filed amendments, and not already on the date of issue of the communication under R. 71(3) EPC. Fees which the appellant had been invited to pay at that earlier stage had not yet become payable and decreased in the same measure that the appellant had reduced the number of claims in response to the communication under R. 71(3) EPC. The board stressed that the EPO could not retain fees of more than insignificant amounts which had been paid for no legal reason.

3. Additional searches during examination

3.1 Additional search: limited discretion of the examining division

In T 1924/07 the board held that the applicant's acknowledgement in the original application that certain prior art is known is in general not a sufficient reason for not carrying out an additional search. The only condition under which an additional search can be dispensed with is where all the technical features of a claim correspond to "notorious" prior art.

C. Special features of opposition and appeal proceedings

1. Transfer of opponent status

1.1 General principles

In T 1911/09 the board held that G 4/88 (OJ EPO 1989, 480), although referring to the business assets and the opposition as being inseparable, did not provide any basis for transfer of opponent status as an obligation or an inevitable consequence of a transfer of business assets related to the opposition. On the contrary, the wording chosen ("transferable or assignable") stressed that the decision rather sought to define conditions under which a transfer of the opposition was possible. Therefore, G 4/88 did not foresee an "automatic" transfer of opponent status in cases of a contractually agreed transfer of business assets.

In order to ensure procedural certainty the case law (T 1137/97, T 1421/05) has established that a contractually agreed transfer of an opponent status has to be explicitly requested and supported by sufficient evidence.

1.2 Evidence for and effect of a transfer

In T 1982/09, it was common ground between the parties that the original opponent (Siemens AG) had transferred its business part "Carrier Network Geschäfts" to Siemens Networks GmbH & Co. KG, which had subsequently changed its name to Nokia Siemens Networks GmbH & Co. KG. For the reasons set out below, the board concluded that the opponent status had not been validly transferred.

In the board's judgement, the declaration submitted by Nokia Siemens Networks GmbH & Co. KG in support of the transfer of opposition did not constitute sufficient evidence in order to conclude that the part of the business transferred comprised all of the technology to which the patent in suit related. More specifically, the term "Carrier Networks" was open to different interpretations and the declaration did not give details of the technological areas covered by the transferred business part. Nor did the further evidence as submitted in reply to the first communication, in which it was confirmed that Siemens AG had transferred "its assets, liabilities and contracts relating to the carrier networks equipment and services business with telecommunication network operators and service providers of Siemens' communications group (COM) comprising: (a) the mobile networks, fixed networks and carrier services divisions; and (b) the carrier core development and supply chain management carrier networks functional divisions", provide sufficient evidence in order to conclude that the transferred business part comprised all of the technology to which the patent in suit related.

The board noted that the patent in suit was concerned with a communication resource allocation method for adapting the transmission capacity of a wireless link between a mobile terminal apparatus and a base station connected to a cellular communication network. Independent claim 9 as granted was directed to a mobile terminal. Hence, the patent related, inter alia, to business activities in the field of mobile telephones. At the oral proceedings before the board, the representative acting for both Siemens AG and Nokia Siemens Networks GmbH & Co. KG did not contest that Siemens AG was currently the applicant for or patent proprietor of patents relating to mobile telephones.

In T 184/11 the board found that not until 14 May 2013, in oral proceedings before the board, was conclusive evidence provided that Fontaine (opponent and respondent) had transferred the opposition, as part of its business operations, to TMGL as from 31 October 2010. Only from 14 May 2013, therefore, could the request for transfer of Fontaine's party status be granted.

The board could not share the appellant's view that, following T 956/03, any transfer occurring before the appeal period expired also had to be substantiated by the end of that period. In T 956/03 the board had followed established case law, i.e. that a transfer could not be acknowledged until appropriate evidence was provided. It had also had to rule on the admissibility of an appeal filed by a party which maintained that it was successor in title to one of the opponents in the first-instance proceedings but failed to provide evidence of that within the appeal period. It was in that light that the principle set out in T 956/03 – that if a transfer occurred before expiry of the appeal period, then the evidence showing that the transferee was entitled to take the opponent's place also had to be submitted by the end of that period – was to be understood.

In the case at issue, however, a party that had been opponent in the first-instance proceedings had initially been respondent in the appeal proceedings, and only during the latter was transfer of party status requested. The board therefore took the view that applying the above principle would not be justified. That was also in line with the case law holding that an opponent's party status can pass to a third party in appeal proceedings even after expiry of the appeal period, irrespective of whether the opponent is appellant or respondent.

2. Intervention

2.1 Admissibility

2.1.1 National infringement proceedings

In T 1713/11 the board stated that the EPC can give no specific definition of what constitutes proceedings for infringement but can only refer to infringement in the broadest of terms. Furthermore, it is clear that intervention was conceived as a procedurally exceptional situation, which is justified only by a substantial legitimate interest of the assumed infringer to enter the opposition proceedings. This substantial legitimate interest does not arise from the fulfilment of special procedural provisions, but from the fact of actually having been confronted with infringement proceedings (or at least having been called upon to cease an alleged infringement seriously enough so as to justify a counteraction).

Given this exceptional nature of the intervention, it does not appear likely that there would have been a legislative intent to create an elaborate and intricate system of procedural provisions for the admissibility of interventions, and therefore the board should also refrain from creating such a system through its jurisprudence. Rather, it appears preferable to concentrate on the examination of the substantive conditions for admitting an intervention, namely whether the action of the proprietor reaches the level which is sufficient to establish a substantive legitimate interest to intervene.

The fulfilment of the requirements of R. 89(1) EPC should be examined similarly, concentrating on the legislative intent instead of creating further procedural preconditions. According to the board, not any step will do. As in the decision T 305/08, the board held that nullity proceedings concerning the patent at issue would not qualify and neither would saisie contrefaçon proceedings. However, it was not apparent to the board why the definition of "proceedings for infringement" should be limited to civil proceedings, to proceedings that allow for certain remedies to be claimed, or two party proceedings. The board therefore concluded that as long as a patent proprietor or any other party entitled to do so initiates proceedings which aim to establish whether a third party is commercially active in an area that falls within the patent proprietor's right to exclude, such proceedings are "proceedings for infringement" within the meaning of Art. 105 EPC.

2.1.2 Proceedings before the EPO

(a) Intervention in appeal proceedings

In T 1961/09 the board decided and stated in the catchword, "While an intervention under Art. 105 EPC shortly before oral proceedings in an appeal, raising new issues, will normally require the oral proceedings to be adjourned if not the remittal of the case altogether (G 1/94, OJ EPO 1994, 787), the oral proceedings may be continued if and so far as this can be done without unfairness to the other parties, in particular the proprietor."

The intervention was based on proceedings which the intervener had instituted against the proprietor in the Portuguese national courts for a declaration of non-infringement following receipt of a cease-and-desist request from the proprietor. On 20 June 2013 notice of intervention was filed. In the course of the oral proceedings on 26 June 2013, the board decided that the intervention was admissible. The proprietor objected to the proceedings being continued because it had not had sufficient opportunity to react to the new position.

After the matter had been discussed with the parties, the board decided that the oral proceedings should be continued, at least with a discussion of an objection raised by the opponent. This was made possible because the intervener agreed that it would not make any submissions on this issue. This meant that the issue could be discussed without causing any unfairness to the proprietor, because the discussion would be limited to an issue already in the proceedings before the notice of intervention was filed. Proceeding in this way was also fair to the opponent, who had a legitimate interest in seeing the proceedings concluded without further delay.

The proprietor then requested that the case be remitted for further consideration of the new issues. The board decided not to remit the case but to continue the proceedings by hearing the opponent's clarity objections against the auxiliary requests then on file. This was again possible because the intervener agreed that it would withdraw all its own clarity objections to the proprietor's auxiliary requests then on file. The board was therefore in a position to hear the arguments of the opponent and the proprietor on these issues of clarity without the intervener adding any further objections.

3. Apportionment of costs

3.1 Abuse of procedure

In T 2165/08 the respondent (opponent) sought an apportionment of costs because the appellant (patent proprietor) filed its auxiliary requests 1 to 3 and document D51 one month before the oral proceedings and document D52 even later.

The requests filed by the appellant with the statement of grounds of appeal in December 2008 were a main and four auxiliary requests which were all unchanged from requests filed before the opposition division.

According to the board it seemed clear that the appellant made a conscious decision, when substantiating its appeal, to pursue the requests which did not succeed at first instance and not to file any additional requests. In doing so it took the double risk that, if it changed its mind later and filed further requests in addition or in substitution, such requests might not only be inadmissible (Art. 13 RPBA) but that, depending on the circumstances, the respondent might seek an apportionment of costs. Auxiliary request 3 was admitted into the proceedings as the respondent did not object to it, but the board held that the other requests filed one month before oral proceedings could, and to comply with Art. 12(2) RPBA should, have been filed with the statement of grounds of appeal and the appellant's behaviour was inequitable.

As regards the late-filed documents, the board stated that the last-minute production of new evidence is not only discouraged by the rules of procedure but also inevitably causes additional work and cost for the party faced with it. In the case at issue, the absence of explanation, both of the evidence itself and of its late production, made matters worse. It would always be unacceptable for a party to produce evidence without any explanation – unless conceivably its relevance was self-evident. But to produce evidence at the virtual end of the proceedings accompanied only by a statement that the explanation will be provided even later was not only discourteous but a clear, and apparently deliberate, attempt to frustrate the respondent's preparation for the oral proceedings.

That was both an act prejudicing the efficient conduct of oral proceedings and an abuse of procedure (Art. 16(1)(c) RPBA and Art. 16(1)(e) RPBA), which merited an apportionment of costs. Therefore the appellant was ordered to pay the respondent's costs incurred by the late-filed requests and documents.

In T 1404/10 the respondent called into question the identity of the appellant (opponent) at the first oral proceedings on 26 April 2013. As a consequence, the oral proceedings were rescheduled with a shorter, six-week period of notice, and the appellant was requested to submit evidence of its identity or of a legal transfer of rights at the latest two weeks before the new date.

The appellant requested an apportionment of costs whereby the respondent would bear the hotel and travel expenses incurred by the head of the appellant's patent department as a result of the rescheduled oral proceedings.

The board noted that the respondent had waited until the oral proceedings to voice its doubts about the identity of the appellant present, which had necessitated the setting of a second date for the proceedings. The respondent, however, had admitted that it had already known about the relevant circumstances four days before the date of the oral proceedings.

The appellant, for its part, owned to having neglected to report the transfer of business operations. The substitution of a party however was only admissible in proceedings if a request to this effect had been filed with the board and evidence of the succession produced, which were steps only the appellant could have taken in this case.

In the board's view, both parties shared responsibility for the need to set a new date for oral proceedings. In particular, the board was unable to find evidence of an abuse of procedure, making it inequitable to order a different apportionment of costs in favour of the appellant under Art. 104(1) EPC 1973. The request for apportionment of costs was therefore refused.

D. Opposition procedure

1. Filing of the opposition and admissibility requirements

In the two parallel cases T 1553/06 and T 2/09 involving the same parties but different patents, the same board dealt with the admissibility of the respective oppositions filed within the framework of a test case. In particular, the board examined whether the parties and their representatives had worked together on a test case in order to obtain answers from the EPO to specific legal questions regarding prior art.

To establish the admissibility of its oppositions, the opponent (appellant) relied basically on the argument that the contentious character of the opposition proceedings was not a general principle and, even if it were, the parties in the case at issue satisfied the criteria for "contentious proceedings". The existing co-operation between the parties' representatives on the test case that arose out of a discussion in a study committee did not render the proceedings non-contentious.

Following the principles set out in G 9/93 (OJ EPO 1994, 891) and G 3/97 (OJ EPO 1999, 245), the board emphasised the contentious nature of opposition proceedings as a necessary condition for the admissibility of the opposition and examined whether an abuse of procedure, i.e. because the opponent was acting on behalf of the patent proprietor ("straw man"), rendered the opposition inadmissible in the case before it. The board could not find a circumvention of the law by abuse of process as it saw no reason to cast doubt on the parties' submissions that the opponent was not bound by any instructions from the patentee or the study committee.

As to the further question whether the opposition proceedings were deprived of their contentious character by the very fact that the parties defended their positions within the framework of a test case, the board did not see any reason to question the contentious nature of the proceedings. An opposition filed within the framework of a test case was not inadmissible for that sole reason, provided that the prosecution of the proceedings thereby instituted was contentious because the parties were defending mainly opposing positions. The respective oppositions were therefore deemed admissible.

2. Identity of the opponent

A natural or legal person filing an opposition must be identifiable at the latest at the end of the opposition period.

In T 22/09 the company in whose name the opposition had been filed had ceased to exist in the year 2005 but was later restored to the register of companies under national law. The opponent company was dissolved before the decision of the opposition division was issued but restored to the register of companies under national law after the filing of the appeal. The restoration had retroactive effect under national law.

The board stated that in principle the question whether a company existed was to be determined by the law under which it had been created. However, the board also stated that although the opposition was admissible when it was filed, the opposition proceedings could have been terminated at any point between 4 October 2005 (on which date the opponent company was dissolved) and 29 October 2008 (the date of the decision under appeal) on the ground that the sole opponent had ceased to exist.

Moreover, since the appellant did not legally exist at the time when the appeal was filed in its name, or at any point during the two-month period referred to in Art. 108 EPC for filing the notice of appeal, the appeal could have been rejected as inadmissible under R. 101(1) EPC on the ground that it did not comply with Art. 107 EPC.

The question was whether those defects could be cured retroactively by virtue of a provision of national law which restored a company to the register seven years after its dissolution and deemed the company never to have been dissolved. The board could find no clear answer to that question in the EPC or in the case law of the boards of appeal.

Since the board could not determine whether there was a proper basis for the continuation of the opposition proceedings by the restored company and whether the appeal was admissible without an answer to the question, it deemed it appropriate to make a referral to the Enlarged Board of Appeal under Art. 112(1)(a) EPC. The board thus referred the following questions to the Enlarged Board of Appeal:

1. Where an opposition is filed by a company which is dissolved before the opposition division issues a decision maintaining the opposed patent in amended form, but that company is subsequently restored to the register of companies under a provision of the national law governing the company, by virtue of which the company is deemed to have continued in existence as if it had not been dissolved, must the European Patent Office recognise the retroactive effect of that provision of national law and allow the opposition proceedings to be continued by the restored company?

2. Where an appeal is filed in the name of the dissolved company against the decision maintaining the patent in amended form, and the restoration of the company to the register of companies, with retroactive effect as described in question 1, takes place after the filing of the appeal and after the expiry of the time limit for filing the appeal under Art. 108 EPC, must the board of appeal treat the appeal as admissible?

3. If either of questions 1 and 2 is answered in the negative, does that mean that the decision of the opposition division maintaining the opposed patent in amended form automatically ceases to have effect, with the result that the patent is to be maintained as granted?

The case is pending as G 1/13.

3. Substantiation of the opposition

3.1 Need to indicate facts and evidence

According to established case law, R. 76(2)(c) EPC does not prescribe a complete "indication of the facts, evidence and arguments presented in support" so as to permit a conclusive examination on that basis alone. For the admissibility of an opposition, the statements made in the opponent's pleadings need not even be true or the arguments conclusive in themselves. It is merely necessary that the opposition division and the patent proprietor are able to follow them.

In T 613/10 notice of opposition was filed in due time. The notice and two additional sheets listed 26 items of evidence, which however were not filed with the EPO until after the opposition period had expired. So did it contain, as required by R. 76(2)(c) EPC, "an indication of the facts … presented in support", i.e. was the opposition sufficiently substantiated?

Two grounds for opposition (lack of novelty, lack of inventive step) were raised, but the notice did not say which citations related to which. None of the cited passages or drawings was compared with the features of any of the 26 claims; nor was any technical connection established between the citations and the claims.

For the board, the opposition's admissibility turned on whether the few indications in the notice plus additional sheets enabled both opposition division and patent proprietor – without further investigation – to understand the gist of the opposition and, in particular, to follow unequivocally the opponent's arguments on at least one of the 26 claims and ground for opposition.

According to the case law, an opposition is adequately substantiated if it gives information about one citation enabling opposition division and patent proprietor to assess the cogency of one of the grounds for opposition raised. Nonetheless, the board held that its admissibility had to be decided in the light of the circumstances of the case, especially if the requirements of R. 76(2)(c) EPC were not clearly and unambiguously met. Those circumstances included not just the technical complexity of the subject-matter in the opposed patent and the citations but also the number of claims the patent in question contained and the number of citations – even if in principle an opposition could contain any number of attacks or citations.

The board ruled that the patent proprietor should not have to try and work out the opponent's case for himself, from inadequate information given in the notice of opposition. For the reasons given above, the board found that the opposition, although filed within the 9-month time limit, did not properly indicate the facts presented in support of either of its two grounds, lack of novelty and lack of inventive step. Thus neither ground had been adequately substantiated, the requirements of R. 76(2)(c) EPC were not fulfilled, and the opposition division had been right to reject the opposition as inadmissible under R. 77(1) EPC. Accordingly, the board dismissed the appeal.

4. Late submission

4.1 Opposition divisions' exercise of discretion

A board of appeal should only overrule the way in which a department of first instance exercised its discretion in a decision in a particular case if the board came to the conclusion that the department of first instance had, in its decision, exercised its discretion according to the wrong principles, or without taking into account the right principles, or in an unreasonable way (G 7/93, OJ EPO 1994, 775). The question to be examined therefore was whether or not the opposition division has exercised its discretion correctly.

In T 1485/08 document (16T), which was an English translation of a Korean patent (document 16), had been submitted by the appellant during oral proceedings before the opposition division and was resubmitted with the statement of grounds of appeal. The opposition division had held that the translation had been filed too late in the proceedings and should therefore be disregarded. This decision was challenged by the appellant on the grounds that the opposition division in disregarding document (16T), which was highly relevant to the issue of novelty, had not properly exercised its discretion. The board stated that an opposition division has a discretionary power under Art. 114(2) EPC to disregard evidence which has not been submitted in time. According to the established jurisprudence of the boards of appeal, a decisive criterion for admitting late-filed documents was their prima facie relevance (T 1002/92, OJ EPO 1995, 605).

In this context the board noted that document (16) was filed with the notice of opposition against novelty of the patent in suit. Although not in one of the official languages of the EPO, from the use of various English terms in the description of document (16) it was already discernible that its subject-matter was related to the claimed subject-matter.

The board was of the opinion that the opposition division in not admitting document (16T) for the sole reason that it was late-filed, without having examined its relevance or considered any other criteria, did not properly exercise its discretion (see also T 418/10).

In T 2165/10 documents E1-E10, which related to an alleged public prior use, were filed by the appellant after expiry of the nine-month opposition period. By applying the criteria of prima facie relevance, they were not admitted in the proceedings by the opposition division. The board shared the opposition division's conclusion that E1-E10 were filed late without any good reason for doing so. There was indeed no change in the file during the opposition proceedings, such as a patent proprietor's new request, which could have justified the late filing. Furthermore, the board considered that the opposition division applied the principle of prima facie relevance correctly in a reasonable manner when exercising its discretional power. Consequently, the board did not see any reason to overturn that decision.

The reasons mentioned by the appellant for the late submission of E1-E10 were that the public prior use had taken place more than 20 years ago in a foreign country. Since the legal time limit for keeping documents had long elapsed, many of the relevant documents had been destroyed. The board, however, shared the respondent's view that the appellant should have included an indication of the alleged public prior use in its notice of opposition and indicated/filed all evidence in its possession at that time. Any further evidence, like the commercial documentation, could have been indicated to be filed later, indicating the difficulties in retrieving such documents in the archives, abroad and/or with third parties.

E. Appeal procedure

1. Filing and admissibility of appeal

1.1 Appealable decisions

1.1.1 Decisions

a) Examples of appealable decisions

If a European patent application is finally deemed to be withdrawn after an admissible appeal against a decision refusing it has been filed, the appeal can usually be considered disposed of, because there is no possibility of a European patent being granted for the application. However, where the sole aim of the appeal is to obtain a finding by the board of appeal that a substantial procedural violation occurred in the first-instance proceedings, such that the appealed decision is to be set aside and the appeal fee reimbursed, the appeal cannot be dealt with in this way. In these circumstances the appellant has a legitimate interest in receiving a decision on the merits of the appeal. Therefore the appeal must be examined and the appeal proceedings cannot be closed without a substantive decision on the case (T 2434/09).

1.2 Board competent to hear a case

In G 1/11 the Enlarged Board of Appeal held that a technical board of appeal was competent to hear an appeal against an EPO examining division's decision – taken separately from its decision granting a patent or refusing the application – not to refund search fees under R. 64(2) EPC.

The board in T 1676/08 found that to comply with Art. 21(4)(b) EPC 1973, it was sufficient for the board to indicate in EPO Form 3303.15 that the complexity of the case was the reason for enlarging the board from three to five members; it was not obliged to give detailed reasoning why it considered the case at issue complex. A change in a board's composition with regard either to the number of its members under Art. 21(4)(b) EPC 1973 or to the replacement of a member under Art. 4 of the Business Distribution Scheme of the Technical Boards of Appeal for the year 2012 (Supplement to OJ EPO 1/2012, 12) was not a decision within the meaning of Art. 106 EPC and Art. 113(1) EPC 1973.

1.3 Entitlement to appeal

1.3.1 Party adversely affected

a) Opponent

The board found in T 1982/09 that the opponent status had not been validly transferred, which meant that the opposition division had thus incorrectly held that company "A" was the new opponent. Consequently, the appeal had been filed in the wrong name. Following T 1178/04 (OJ EPO 2008, 80), the fact that the opposition division's ruling on the issue of opponent status was wrong could not mean that company "A" was not a party to the proceedings at the date the notice of appeal was filed. A person is to be regarded as a party for the purposes of Art. 107 EPC even if his entitlement to take part in the proceedings is brought into question and such entitlement is the subject-matter of a pending decision. Although he may cease to be a party if he is held not entitled to take part in the proceedings, this does not mean he was never a party. The appeal was admissible.

1.4 Form and time limit of appeal

1.4.1 Form and content of notice of appeal (Rule 99(1) EPC)

a) Rule 99(1)(a) EPC

Where there is objectively a deficiency in the notice of appeal indicating a genuine error as regards the identity of the appellant and there is objective evidence in the file indicating with a sufficient degree of probability who the appellant should be, then the notice of appeal may be corrected under R. 101(2) EPC (T 1961/09, following T 97/98, OJ EPO 2002, 183). The referral to the Enlarged Board in G 1/12 by the board in T 445/08 (OJ EPO 2012, 588) did not bring into doubt the jurisdiction to correct an error in the notice of appeal in circumstances such as those in T 97/98.

1.5 Statement of grounds

1.5.1 General principles

a) Legal or factual reasons to be specified

In J 10/11 the board reviewed the case law of the boards of appeal on the requirements to be fulfilled in the statement of grounds. If the appellant submits that the decision under appeal is incorrect, then the statement setting out the grounds of appeal must enable the board to understand immediately why the decision is alleged to be incorrect and on what facts the appellant bases its arguments, without first having to make investigations of its own (see T 220/83, OJ EPO 1986, 249, and T 177/97; affirmed by numerous decisions, and in particular recently by T 573/09).

Whether the requirements of Art. 108, third sentence, EPC in conjunction with R. 99(2) EPC are met has to be decided on the basis of the statement of grounds of appeal and of the reasons given in the contested decision (see e.g. J 22/86, OJ EPO 1987, 280; T 162/97). Exceptionally, it has been acknowledged that the requirement for admissibility may be regarded as satisfied if it is immediately apparent upon reading the decision under appeal and the written statement of grounds that the decision should be set aside (see J 22/86).

The appellant had made no submissions regarding the causal relationship between the reasons given in the statement of grounds of appeal and the asserted invalidity of the findings of the decision impugned. If no causal relationship were required, any submission, even one not connected with the reasons on which the decision impugned is based, would be acceptable. This would render the provisions of Art. 108 EPC moot. Whilst the grounds do not have to be conclusive in themselves, i.e. justify the setting aside of the decision impugned, they must enable the board to assess whether or not the decision is incorrect. The appeal was rejected as inadmissible.

In T 395/12 the appeal was also held inadmissible; the applicant's only statement that directly addressed the decision under appeal was that the examining division was "wrong", with no explanation why. Decisions T 213/85 (OJ EPO 1987, 482) and T 95/10 clarify that the appeal procedure is not a mere continuation of the examination procedure (in accordance with decisions G 10/91, OJ EPO 1993, 420; G 9/92, OJ EPO 1994, 875 and G 4/93, OJ EPO 1994, 875), but separate therefrom. Where the applicant in the grounds of appeal repeats its arguments set out during the examination phase without taking into account the decision under appeal, it mistakes the function of the boards of appeal; they are not a second go at the examination procedure, but are meant to review decisions made by the examining divisions, based on the objections raised against the decision in the grounds of appeal, which must therefore relate to the reasons on which the decision under appeal is based.

The appeal had also to be considered inadmissible because the grounds failed to deal with all the reasons the examination division advanced for refusing the application. According to T 213/85 and T 1045/02, the grounds of appeal must deal with all those reasons on which the decision under appeal is based. This is consistent with the requirement of Art. 12(2) RPBA, according to which, "The statement of grounds of appeal and the reply shall contain a party's complete case".

See also T 473/09, where the appeal was also held inadmissible as the grounds failed to deal with all the reasons for refusing the application.

In T 2532/11 the appeal was found inadmissible as the appellant's statement of grounds did not contain any reference to the impugned decision, let alone any explanation as to why it was wrong and should be set aside (see also point 1.5.2 below).

1.5.2 New case raised

a) By patent proprietor or applicant and appellant

In T 2532/11 the question also arose whether newly filed requests could be seen as implicit grounds of appeal. A statement of grounds of appeal supported by amended claims may define, at least implicitly, the extent to which the appellant wishes the decision under appeal to be set aside. Many decisions have adopted a lenient position and deemed appeals admissible if the competent board was able to infer from the particulars of the case the presumed intentions of the appellant and the probable reasons underlying its actions (T 729/90, T 563/91, T 574/91, T 162/97). Appeals have also been held admissible where there was a change in the subject of the proceedings due to the filing of new claims together with the statement of grounds and the latter set out in detail why the raised grounds for opposition did not prejudice the maintenance of the patent as amended on the basis of these new claims (T 717/01, T 934/02 referring to J xx/87, OJ EPO 1988, 323 and T 105/87).

According to the board, the purpose of the statement of grounds, together with the notice of appeal is to define the scope of the appeal. This definition lies within the discretion of the appellants as a part of the principle of free disposition.

Without prejudice to the provisions of Art. 114 EPC, which are of limited application in opposition proceedings, the board of appeal has the duty to assess whether the appeal is well-founded within the frame of the case as presented by the appellants but it cannot guess what the arguments are, let alone, provide arguments in lieu of the appellants. Conversely, the principle of free disposition does not extend such that it should allow the appellants to build a new case, so as to render the decision under appeal purposeless. A direct link must be maintained between the decision under appeal and the statement of grounds of appeal. Whilst the appellant should not be denied the right to file amended claims, it had to provide arguments in order to explain which issue(s) in the decision it considered erroneous and provide arguments and evidence in support.

In T 933/09 the appeal was also rejected as inadmissible; the board considered that the mere filing of amended claims did not exonerate the appellant from the task of expressly specifying in the statement of grounds of appeal the relevance of the amendments for overcoming the objections on which the decision under appeal was based.

1.5.3 Exceptional circumstances warranting admissibility of the appeal

a) Substantial procedural violation

In T 1020/13 the board held that, if a substantial procedural violation is alleged and adequately substantiated in the statement of grounds of appeal, it is irrelevant for the decision on admissibility of the appeal whether the challenge to the substantive reasons for the decision can likewise be regarded as adequately substantiated.

1.6 Admissibility of appeal to be checked in every phase of appeal proceedings

In T 670/09, the respondent had not challenged the appeal's admissibility until two days before the oral proceedings.

The board observed that, under Art. 12(2) RPBA, the reply to an appeal had to contain the respondent's entire case. As a rule, respondents had to object to the appeal's admissibility or the board's competence in limine litis, i.e. before submitting their substantive defence. They were not obliged to object to admissibility and could by all means confine their reply under Art. 12(2) RPBA to a substantive defence but, if they did so, they consented to the board's investigating the related facts. The respondent's objection to admissibility was therefore disregarded.

1.7 Interlocutory revision

According to T 1060/13 it is established case law of the boards of appeal that, in the event that the appeal is objectively to be considered as admissible and well-founded, the department of first instance is obliged to grant interlocutory revision (T 139/87, OJ EPO 1990, 68; T 180/95; T 2528/12); there is no room for discretion in the interests of procedural efficiency (G 3/03, OJ EPO 2005, 344; J 32/95, OJ EPO 1999, 713; T 919/95).

An appeal is to be considered "well-founded" if at least the main request submitted with the appeal includes amendments which clearly meet the objections on which the decision relies, such that the first-instance department could reasonably be expected to recognise this and thus rectify its decision. That there are other objections which have not been removed but which were not the subject of the contested decision cannot preclude the application of Art. 109(1) EPC 1973 (T 139/87; T 47/90, OJ EPO 1991, 486; T 219/93; T 919/95).

Thus, even if the amendments raise new objections not yet discussed, interlocutory revision must be allowed since an applicant should have the right to examination at two instances. Objections or remarks made in an obiter dictum of a decision under appeal cannot be taken into account (see e.g. T 1640/06 and T 726/10; the board disagreed with T 1034/11); the board pointed out that there were inconsistencies between the Guidelines and the established case law in this respect (Guidelines E-X, 7.4.2 – also in the current September 2013 version).

Refusing interlocutory revision in the case of an admissible and well-founded appeal constitutes a breach of the duty to grant interlocutory revision in such a case and thus a breach of the principle of procedural efficiency (contrary to the finding in T 704/05) and cannot be regarded as a substantial procedural violation within the meaning of R. 103 EPC (see e.g. T 794/95).

1.8 Accelerated proceedings

The appellant/patentee in T 734/12 requested acceleration of the appeal proceedings on the basis of three arguments: the possibility of remittal; the suggestion that the decision under appeal, which turned on novelty, related to an important point of law the resolution of which might take additional time; and the commercial and medical importance of the patented and approved treatment.

In the board's opinion these reasons did not in themselves justify acceleration. Many appeals arose from decisions on one point only and thus included the issue whether or not to remit if successful.

Nor was the board satisfied there was an important point of law but, even if there were, this would not necessarily require additional time, there being already considerable case law on the topic. Finally, many if not all patentees considered their patents to be of commercial significance and, if that were a criterion for acceleration, it would be impossible for the board to decide which were of such importance as to merit acceleration and which not. That was doubtless one reason why acceleration had been acknowledged as appropriate when infringement proceedings were threatened or pending, which was not the case here.

However, it was clear there was a substantial measure of agreement between the parties on an early resolution of the issues, and the public had an interest in the early resolution of disputes as to the existence or extent of a patent. The interests of the parties and of the public would therefore best be served by an expedited procedure.

2. Substantive examination of the appeal

2.1 Binding effect of requests – no reformatio in peius

According to T 1843/09 (OJ EPO 2013, 502) the prohibition of reformatio in peius as far as it entails a procedural limitation of the proprietor's liberty to change by way of amendments the scope of protection sought prevails "in principle" (G 4/93, OJ EPO 1994, 875) until the final settlement of the opposition case and, therefore, also in any proceedings, including further appeal proceedings, subsequent to a remittal under Art. 111 EPC.

The board went on to say that it is clear from G 1/99 (OJ EPO 2001, 381) that exceptions to the principle of the prohibition of reformatio in peius are a matter of equity in order to protect the non-appealing proprietor against procedural discrimination in circumstances where that prohibition would impair the legitimate defence of its patent. Therefore, exceptions to the prohibition of reformatio in peius are not limited to the situation specifically dealt with in G 1/99. Rather, the equity approach as taken by the Enlarged Board of Appeal covers, beyond an error of judgment by the opposition division, any change of the factual and/or legal basis on which limitations have been made by the proprietor prior to the appeal by the opponent as the sole appellant, provided the proprietor would be prevented by the prohibition on reformatio in peius from adequately defending its patent against new facts and objections introduced into the proceedings at the appeal stage.

In the appeal proceedings in T 1979/11 the patent proprietor (respondent) submitted a new claim 1 which was broader than the claim 1 as allowed by the opposition division. The opponent (appellant) took the view that, due to this broadening, the new main request violated the principle of the prohibition of reformatio in peius.

The board disagreed. Whilst acknowledging that the broadening of a claim effected by a proprietor (respondent) in appeal proceedings is generally contrary to the principle of the prohibition of reformatio in peius, in the case before it the broadening of claim 1 was a reaction by the respondent to the appellant's objection under Art. 83 EPC, which was raised for the first time in the statement of grounds of appeal.

As outlined in G 1/99, it would not be equitable to allow the appellant (opponent) to present a new attack and at the same time to deprive the proprietor (respondent) of a means of defence. Even though G 1/99 specifically addressed a reaction of the proprietor to an error of judgement by the opposition division concerning the allowability of an amendment, the equity approach as outlined by the Enlarged Board of Appeal is not limited to the situation specifically dealt with in G 1/99, but also covers other situations involving a change of the factual and/or legal basis on which limitations have been made by the proprietor prior to the appeal by the opponent as the sole appellant (see T 1843/09). The board therefore decided to admit the main request into the proceedings.

In T 61/10 the board pointed out that the existence of a causal link between the limiting feature to be deleted and the new situation arising on appeal was a necessary precondition for justifying an exception to the principle of prohibition of reformatio in peius for reasons of equity. In G 1/99, T 1843/09 and T 1979/11 this condition was fulfilled. However, in the case before the board, the priority issue which arose was related to one equally relevant for other claims, independently of the limiting feature. Similarly, the documents cited by the appellant to reinforce an inventive step objection were equally valid for a method of production with or without the limiting feature. As the precondition was not fulfilled, a departure from the principle of prohibition of reformatio in peius based on analogies with the cases cited was not justified.

2.2 Subject-matter under examination

The appellant (patent proprietor) in T 226/09 had requested in the notice of appeal that the contested decision be set aside and the patent maintained in accordance with the first auxiliary request. In the statement of grounds, it gave reasons for that request but also filed a more extensive main request for rejection of the opposition.

In the opponent's view, the subject-matter of the appeal had been limited to the first auxiliary request, so that the new main request was not at issue in the proceedings. The board disagreed. According to the boards' case law, the amended wording of R. 99 EPC had not entailed any substantive revision of the requirements to be met by the notice of appeal and statement of grounds of appeal. The R. 99(1)(c) EPC requirements were met if it was requested in the notice that the contested decision be set aside.

The subject of the appeal always had to be established in the light of the content of the contested decision. Since, where a decision ruled on several matters, the appellant might not wish to contest all those rulings, he could choose to appeal against the decision either as a whole or in part. However, in the latter case, the contested part had to be distinct. In the case in hand, there were no legally distinct parts, only a single coherent decision on the validity of the patent. Given that the appeal had been filed against that decision, the subject of the appeal was clear. To what extent amendment of the contested decision was sought did not have to be indicated until the statement of grounds. Any indication already contained in the notice of appeal was merely anticipatory and had no limiting effect on the grounds.

In T 1799/08 the opponents had ceased to take an active part in the appeal proceedings. The board referred to G 10/91 (OJ EPO 1993, 420), where it was held that the purpose of the appeal procedure inter partes is mainly to give the losing party a possibility to challenge the decision of the opposition division on its merits.

Moreover, the appeal procedure inter partes is, in contrast to the merely administrative character of the opposition procedure, considered as a judicial procedure, which by its very nature is less investigative than an administrative procedure and in which opposing parties should be given equally fair treatment. It was therefore up to the opponents to convince the board that the contested decision to maintain the patent in amended form was incorrect, and that the patentability requirements were not met.

As indicated in G 8/91 (OJ EPO 1993, 346 and 478), the public interest in the European patent system is primarily safeguarded by the possibility of filing an opposition. As a matter of general principle it is not the function of the boards of appeal to carry out a general review of decisions at first instance, regardless of whether such a review has been sought by the parties. In view of the character of the inter partes appeal proceedings, it could not be expected that the board, independently of its preliminary opinion on some of the critical issues for deciding on a ground not properly substantiated by the opponent which had initially raised it, would fully investigate that ground, in breach of the principle of equal treatment of the parties, and provide its own elaborate and full reasoning, substituting itself for that opponent which had remained passive. The board thus accepted, in the absence of any evidence to the contrary, the patent proprietors' arguments.

2.3 Review of first-instance discretionary decisions

Under Art. 106(2) EPC, a decision which does not terminate proceedings as regards one of the parties can only be appealed together with the final decision, unless the first decision allows a separate appeal.

It is undisputed that the boards must respect the margin of discretion conferred on departments of first instance required under the EPC to take decisions at their discretion. They can therefore overrule such a department's exercise of discretion only if they consider that it has exceeded the proper limits of that discretion (see also G 7/93, OJ EPO 1994, 775).

In T 1849/12, the contested communication of the examining division, which had to be regarded as a decision, had not terminated the proceedings as regards the appellant. In its communication, the examining division had considered that an applicant was not entitled to a communication under R. 71(3) EPC so long as examination of the application had not yet been completed, expressly refusing the appellant's auxiliary request for an appealable decision on the ground that there was no suitable legal basis. In doing so, it had clearly failed to take properly into account that, were the appellant to appeal successfully following the final grant decision, there would no longer be any way of sensibly meeting its request for immediate issue of a communication under R. 71(3) EPC to allow for a prompt grant. The contested communication had thus created a grievance which could not be remedied by an appeal following the final decision. The examining division therefore ought to have allowed a separate appeal, especially since the EPC made no provision for an appeal against inaction of the EPO (see, to that effect, also J 26/87, OJ EPO 1989, 329).

The examining division had failed in its deliberations to take proper account of a crucial factor and so exercised its discretion improperly. Its communication therefore had to be regarded as a separately appealable decision.

3. Termination of appeal proceedings

3.1 Withdrawal of the appeal

3.1.1 Board's duty to issue decision when appeal is withdrawn after the decision is announced.

After the board in T 1518/11 had announced its decision to dismiss the appeal against the refusal of the application in suit and closed the oral proceedings, the appellant stated by letter that the appeal was withdrawn. However, since the decision had already been announced at the oral proceedings and thereby became effective on that day, the appeal proceedings had already terminated (following T 843/91, OJ EPO 1994, 818) and the appellant's submission made after the announcement of the board's decision was without any legal effect.

Furthermore, a statement of withdrawal of appeal made by the (sole) appellant after the final decision of the board had been announced at oral proceedings did not relieve the board of its duty to issue the decision in writing, setting out the reasons for the decision, and notify the appellant of it (following T 1033/04).

4. Remittal to the department of first instance

4.1 Remittal following filing or late submission of relevant new document

In T 78/11, the respondent (patent proprietor) had requested remittal of the case, whereas the appellant and the interveners had asked for a final decision. The board cited G 1/94 (OJ EPO 1994, 787), in which it had been held that, where new grounds for opposition were exceptionally admitted, the case should be remitted to the department of first instance for further prosecution, unless there were special reasons for doing otherwise.

It was therefore at the board's discretion whether to remit the case to the department which had taken the contested decision. According to G 1/94, an intervener could admissibly raise new grounds of appeal, meaning that, essentially, intervention was treated as equivalent to the filing of an independent opposition. The nature of the newly submitted evidence was thus just as crucial for the exercise of discretion as the current state of the proceedings.

The board noted that a pending infringement action against the intervener was often the reason behind an intervention, and that this had been a point considered by the Enlarged Board in G 1/94 in finding that, as a rule, cases involving an intervention should be remitted. The board therefore took the view that not too much weight should be attached in this case to the interest of the appellant and, in particular, that of the interveners in avoiding a remittal in view of the pending infringement action. Moreover, the material in the newly submitted documents created a new case. The board also observed that it had been in the interveners' hands to influence the board's decision on remittal through their choice of new evidence (T 1469/07). The case was remitted to the department of first instance for further prosecution.

4.2 Binding effect of decision remitting case to department of first instance

4.2.1 Department of first instance bound by decision of board of appeal

Under Art. 111(2) EPC the EPO department of first instance is bound by the ratio decidendi of the board of appeal if the case is remitted to the department whose decision was appealed. Thus, when after remittal the proprietor files new requests which require examination to be re-opened on issues that have already been judged upon by the board of appeal, without the justification that might be provided, for example, by the proprietor's being faced with a new situation, then such requests should be deemed inadmissible (T 383/11).

5. Late submission in appeal proceedings

The Rules of Procedure of the Boards of Appeal give general directions on how the boards of appeal should exercise their power to admit or disregard material filed at different stages of the appeal proceedings.

5.1 Complete case (Article 12(2) RPBA)

In T 1544/08 the board stated in its catchword that the wish to avoid giving commercially valuable information to competitors was not necessarily a valid reason for not complying with the requirement of Art. 12(2) RPBA that the grounds of appeal should contain the appellant's complete case.

The appellant filed, in support of his objection under Art. 100(b) EPC, modelling results only slightly more than two months before the oral proceedings. The appellant stated that the results presented in this document had been produced on 18 January 2010, but that they had not been filed at that stage of the procedure because of concerns that they might provide technical information which would have been of use to competitors. In the meantime these concerns had become less significant, in which respect the appellant referred to its commercialisation in December 2011 of a product under the name "ClimaCoat". The board was of the opinion that a purely commercial consideration could not represent a valid reason for not complying with the requirement of Art. 12(2) RPBA. The filing of these modelling results only slightly more than two months before the oral proceedings, and not (for instance) sooner after the start of commercialisation of the appellant's product, gave the respondent little time to carry out modelling in response to these submissions. The board also noted that it should have been possible for the appellant to select parameters for the modelling which were unrelated to their product so as to avoid the competition issues. The board therefore considered it appropriate to make use of its discretion under Art. 13(1) and (3) RPBA not to admit the modelling results into the procedure.

5.2 Submissions not examined at first instance (Article 12(4) RPBA)

Art. 12(4) RPBA confers on the boards of appeal discretion to hold inadmissible in appeal proceedings facts, evidence and requests which could have been presented or were not admitted in the first-instance proceedings.

5.2.1 Evidence withheld at first instance

In J 5/11 the Legal Board stated that the Office's duty to examine evidence of its own motion is more stringent in relation to evidence that is in the public domain, and the case for admitting such evidence when it is filed out of time by the parties is correspondingly stronger. The Office's duty to examine of its own motion evidence that is in the private sphere of the party concerned is obviously limited. Such evidence can only be taken into consideration by the Office if it is brought to the Office's notice by the party concerned. If evidence of that type is not put forward in the proceedings before the department of first instance of the Office, it is difficult to see any compelling reason why the board of appeal should exercise its discretionary power under Art. 114(2) EPC and Art. 12(4) RPBA in such a way as to admit the evidence when it is filed with the grounds of appeal or a fortiori at a later stage of the appeal proceedings. That is particularly true when, as in the case at issue, the department of first instance has expressly drawn the appellant's attention to the need for supporting evidence, indicated precisely what type of evidence is needed and given the appellant an adequate time limit within which to file the evidence. Thus, the applicant for re-establishment of rights who failed to substantiate his application adequately in the first-instance proceedings, in particular after being expressly invited to do so, cannot normally make good that failure by submitting additional evidence with the grounds of appeal.

In T 724/08 the board observed that it had the power under Art. 12(4) RPBA to refuse to admit into the proceedings documents D10/D10a and D11/D11a, which the appellant (opponent) had cited as novelty-destroying for the first time in the statement setting out the grounds of appeal, if that evidence could have been presented in the first-instance proceedings.

The appellant had already raised a novelty objection in its opposition and therefore could have submitted these documents during the first-instance proceedings or even during the period for opposition. It was immaterial whether the appellant had indeed only come across these Japanese patent specifications or the corresponding Patent Abstracts of Japan by chance later or whether said documents had genuinely been difficult to find, because it was not permissible to use these issues to slow down proceedings or gain an unfair advantage over the other party. In the same way as a patent proprietor was required to submit amended claims during first-instance opposition proceedings, an opponent was likewise required to submit citations against granted claims at first instance. However if, as in the present case, it first did so in the statement setting out the grounds for appeal, there was a risk that the board would exercise its powers under Art. 12(4) RPBA and refuse to admit the citations in question into the appeal proceedings.

In exercising this power, the board could make admitting a citation into appeal proceedings dependent on whether it is prima facie relevant but the board was not obliged to do so, because otherwise an opponent could easily submit a (highly) relevant citation for the first time in the statement setting out the grounds of appeal and expect the citation to be admitted into the appeal proceedings on grounds of relevance.

In the case before it, the board left open the issue, raised by the parties, of the relevance of the citations because it did not consider it justified on grounds of procedural economy to admit into the proceedings citations that, even if relevant, had been found merely by chance and had not been the target of a selective search at an earlier stage of the proceedings.

5.2.2 Requests withheld by the proprietor in opposition proceedings

In decision T 1125/10 the appellant became aware of the two new documents submitted by the opponent (respondent) and the supporting arguments (at least) three days before the date set for the oral proceedings before the opposition division. It could therefore have reacted in a number of ways, e.g.

  • asked for the late-filed documents not to be admitted;
  • asked for an adjournment of the oral proceedings or
  • reconsidered its decision not to attend the oral proceedings.

None of these options was chosen. No mention of this failure to react was made in the statement of grounds of appeal. However, the issue was not the minimum period to be granted, nor the amount of time the appellant would have needed to prepare for the oral proceedings, which it had anyway not planned to attend. Rather, in deciding whether to exercise its power to hold inadmissible requests first submitted in appeal proceedings, what the board had to determine was whether the fact that the new requests had not already been examined at first instance was down to the department of first instance (in this case the opposition division) or whether the conduct of the party concerned (in this case the appellant) had in fact prevented such an examination (as had been found in T 1067/08).

The board held that it was the appellant's total inaction in the face of obviously relevant new documents which had prevented the opposition division from taking a reasoned decision on the subject-matter of the amended claims based on those documents, meaning that, even though the appellant had again chosen not to attend the proceedings, it now had to decide on the matter at first and final instance or remit the case to the department of first instance. The board therefore decided not to admit the appellant's new requests into the proceedings.

In T 936/09 the board had to decide whether the appellant's (proprietor's) sole request, filed for the first time with the statement of grounds, was admissible in accordance with Art. 12(4) RPBA. In the first-instance proceedings, the proprietor had neither responded in substance to the opposition, nor requested oral proceedings, but had merely stated that it was interested in maintaining the patent as granted. The opposition division had revoked the patent without prior oral proceedings on grounds of lack of novelty. The board stated that under the EPC, there is no legal obligation for the patent proprietor to take an active part in opposition proceedings. However, the patent proprietor is not free to present or complete his case at any time that he wishes during the opposition or opposition appeal proceedings, depending, for example, on his procedural strategy or his financial situation. If a patent proprietor chooses not to respond in substance at all to the opposition, for example by filing arguments or amended claims, or chooses not to complete his submissions at the stage of the first-instance proceedings, but rather presents or completes his case only in the notice of appeal or the statement setting out the grounds of appeal, then he will need to face the prospect of being held to account for such conduct by the board when, for example, exercising its discretion under Art. 12(4) RPBA. This applies in particular if all the reasons for revocation of the opposed patent were known to the patent proprietor before it received the impugned decision.

5.2.3 Requests admitted as a normal procedural development

In T 134/11 the respondent requested that the main request should not be admitted into the proceedings pursuant to Art. 12(4) RPBA, because it could already have been filed in the first-instance opposition proceedings. The board stated that according to Art. 12(4) RPBA, it is in its discretionary power to hold (or not to hold) inadmissible a request which could have been presented in the first-instance proceedings. Consequently, the mere fact that a request could have been filed in the first-instance proceedings as such does not lead automatically to the inadmissibility of this request (as in this case there would be no discretionary power of the board). On the contrary, normally such a request is inadmissible only in exceptional circumstances. For example, such circumstances may arise where, by the filing of a request only at the appeal stage, a decision by the opposition division on certain issues is avoided and the decision is shifted to the second instance (this is referred to as "forum shopping" in decision T 1067/08). In the case at issue it was credible that it was not the appellant's intention to avoid any decision of the opposition division on the current main request. Furthermore the current main request did not raise any new issues but represented merely a continuation of the approach chosen by the appellant all through the first-instance opposition proceedings. Therefore, it could not be argued that by means of filing this request only in the appeal a decision of the opposition division on certain issues had been avoided such that forum shopping between instances had occurred. The main request thus was admissible. Contrary to the respondent's view, this finding was not at variance with decisions T 144/09 and T 936/09. The reason for not admitting certain requests in these decisions was essentially that the proprietor had not filed any request in the first-instance opposition proceedings which could have overcome the opposition division's objection and in fact had done so only in the appeal.

5.2.4 Review proceedings

In both R 11/11 and R 13/11 the respective petitioners complained that the board of appeal decided in its discretion under Art. 12(4) RPBA not to admit the petitioner's auxiliary requests. The Enlarged Board held that its case law clearly showed that petition proceedings may not be used to review the exercise by a board of appeal of a discretionary power if that would involve an impermissible consideration of substantive issues. This has also been confirmed in the specific context of the discretion in Art. 12(4) RPBA.

In R 4/13 the Enlarged Board stated that in cases where the discretion not to admit requests is exercised by a board in reliance on Art. 12(4) RPBA it is inevitably a matter of speculation whether and in what circumstances the party would have filed such requests. Such speculation is however irrelevant as regards the existence and exercise of the discretion and the board quite correctly did not enter into such speculation. In reality it was not therefore in dispute that the board had jurisdiction not to admit the requests. The point about the admittance of these requests was raised in the board's communication and the petitioner was heard on it at the oral proceedings.

5.3 Amendments to a party's case – procedural economy (Article 13(1) RPBA)

In T 1033/10 the complexity of the amendment was not found to be an issue which by itself would have led the board not to admit the request which had been filed at 14.40 hrs at the oral proceedings. The board stated that there had been plenty of occasions for the appellant to amend the claims in reply to the clarity objections. Although the appellant argued that it had understood the objection as regards lack of clarity only as a result of the debate during oral proceedings, this did not justify filing a request at that stage of proceedings to deal with the objection.

The state of the proceedings and the need for procedural economy taken together imply a requirement on a party to present appropriate requests as soon as possible if such requests are to be admitted and considered. If an objection (made by a party and/or by the board) is not fully understood in a particular case, it is then, at the very least, incumbent on the party having difficulties in understanding an objection to indicate this at the earliest possible stage and make the appropriate effort to have the objection clarified. A lack of understanding alone cannot justify postponing amendments to a party's case until a later stage in the proceedings.

In T 1732/10 the board held that not reacting in substance to the appeal of the opponent, but waiting for the board's preliminary opinion before any substantive reaction is filed, is regarded as an abuse of procedure. It is contrary to the equal distribution of rights and obligations upon both sides in inter partes proceedings and to the principle that both sides should set out their complete case at the outset of the proceedings. Both principles are clearly established by the Rules of Procedure of the Boards of Appeal.

This is all the more so if the substantiation for all the requests, which were filed after summons to oral proceedings have been sent, is filed only shortly before the oral proceedings before the board. Such requests – which are not self-explanatory – are considered by the board as submitted only on the date of their substantiation. Such very late requests are contrary to procedural economy, do not take account of the state of the proceedings and cannot be reasonably dealt with by the board without adjournment of the proceedings or remittal to the department of first instance, contrary to Art. 13(1) and 13(3) RPBA.

Where such very late requests take up subject-matter only available from the description, it cannot automatically be assumed that it was covered by the initial search, nor that it is automatically the responsibility of the opponent to perform such a search.

5.4 Absence of a party from oral proceedings (Article 15(3) RPBA)

In T 1949/09 the board did not admit the late-filed tests into the proceedings. The only justification put forward for the late filing of the tests, namely the time needed to have the tests authorised and performed, could not justify their filing almost three years after the grounds of appeal, more than two years after the reply of the respondent and only 15 days before the convened oral proceedings, all the more so as the appellant never informed the board of any difficulty in providing further evidence, nor requested additional time to prepare such evidence. Apart from the unjustified, late filing, the tests raised a number of questions about their significance, which could not be answered without an adjournment of the proceedings. Art. 15(3) RPBA, which provides that the board is not obliged to delay any step in the proceedings, including its decision, by reason only of the absence at the oral proceedings of any party, does not go so far as to nullify Art. 113(1) EPC. On that basis, the respondent, who chose not to attend the oral proceedings, could rely on the procedural provisions, namely that the decision would be made on the basis of the written case within the meaning of Art. 12 RPBA and that no significant amendment of the case would be admitted which would go beyond what is provided by Art. 13 RPBA (see also T 1621/09).

6. Reimbursement of appeal fees

In T 1500/10 the board held that if oral proceedings took place at the instance of the EPO because it considered this to be expedient under Art. 116(1) EPC and the duly summoned party did not attend them without serious reasons, this conduct might have the consequence that it would not be equitable to reimburse the appeal fee. (See also under Chapter III.C. "Oral proceedings".)

In relation to the reimbursement of the appeal fee, see also the pending referrals G 1/14 (referring decision T 1553/13) and G 2/14 (referring decision T 2017/12).

7. Proceedings before the Enlarged Board of Appeal

7.1 Important points of law

7.1.1 Situations where referral is not appropriate

The appellant in T 2459/12 requested referral to the Enlarged Board of Appeal of questions concerning the correct application of R. 164 EPC, as questions raising a point of law of fundamental importance. The board acknowledged that the proposed questions would have a bearing on the outcome of the case, but referred to T 26/88, where a request for referral was rejected as the Implementing Regulations had been subsequently amended. As the problem was therefore likely to arise very rarely in future, the question was not sufficiently important to justify a referral.

A version of R. 164(1) EPC which the appellant accepted would remove the disadvantageous effects of the current version had been adopted by the Administrative Council. There was thus no significant general interest in having the referred questions answered, as the decision of the Enlarged Board would affect only a relatively small number of applicants for a limited period of time, after which it would become obsolete. Nor was it reasonable to imagine that the decision of the Enlarged Board would be issued before the coming into force of the new rule. Furthermore, the board had reached its decision based on what it believed to be the correct interpretation of opinion G 2/92 (OJ EPO 1993, 591), so no referral was necessary in order to decide the case.

7.2 Petition for review under Article 112a EPC

7.2.1 General

The petition in R 15/13 was written in a highly emotive style and included a large volume of derogatory language, much of which (for example, "fabricating the decision", "partiality", "stolen patent", "jealousy and outright bad faith", "aggressive and corrupt", and "fraud") was excessive and without apparent basis in the petition.

The Enlarged Board considered accordingly whether the petition was "in a reasoned statement" as required by Art. 112a(4) EPC. However, in view of the representative's written apology for his use of such language and the actual withdrawal of the offensive allegations at the commencement of the oral proceedings, the Enlarged Board did not pursue this further. The petition was found admissible. (It was ultimately rejected as clearly unallowable.)

7.2.2 Scope of review by the Enlarged Board of Appeal

a) Extent of scrutiny by the Enlarged Board of Appeal

It has again been confirmed that a petition for review cannot be used as a means of reviewing the way in which a judicial body such as a board of appeal has exercised its discretion in relation to a procedural matter (R 4/13, following R 10/09 and R 11/11).

7.2.3 Obligation to raise objections in accordance with Rule 106 EPC

a) Obligation to raise objection

The Enlarged Board in R 5/12, referring to R 4/08, pointed out that raising an objection pursuant to R. 106 EPC was a procedural act and, except where the objection could not be raised during the appeal proceedings, a precondition for access to the extraordinary legal remedy against final decisions of the boards of appeal. Its validity depended on compliance with the following two criteria:

  • the objection had to be expressed in such a form that the board could recognise immediately and without any doubt that an objection pursuant to R. 106 EPC was intended, and
  • the objection had to be specific, i.e. the party had to indicate unambiguously which of the defects listed in Art. 112a(2)(a) to (c) and R. 104 EPC it wished to invoke.

In T 1544/08 the board re-opened the debate after the appellant raised an objection under R. 106 EPC that the board had committed a substantial procedural violation in 'deciding' that the figures of the application as originally filed were those filed by the respondent. Following the further discussion, the board indicated that it considered that the procedural objections raised by the appellant had been overcome.

b) Objections not complying with Rule 106 EPC

In R 2/12 the Enlarged Board highlighted just how much the formal requirement of an objection under R. 106 EPC was justified (as opposed to a mere aside); it is additional to and distinct from other statements (following R 4/08). It is the duty of a party to check whether its objection to a fundamental procedural defect occurring during oral proceedings has been recognised by the board and will be dealt with. In the case at issue there had been disagreement between the parties and the board about just what had been said. As the procedural defect was apparent during the oral proceedings, the petition was held inadmissible under R. 106 EPC.

In T 518/10 the appellant raised an objection under R. 106 EPC, alleging that its right to be heard under Art. 113(1) EPC had been violated in that its request for a postponement of oral proceedings had been refused and certain documents not admitted. The board was aware that R. 106 EPC laid down an admissibility requirement for a petition for review the examination of which was within the jurisdiction of the Enlarged Board of Appeal. Nevertheless, the point in time at which an objection is raised affected the alternatives of responses available to the board concerned. The board should be able to react immediately and appropriately by either removing the cause of the objection or dismissing it (R 4/08, R 14/11). Therefore the appellant should have raised any objection to the refusal to postpone the oral proceedings immediately after notification of the relevant communication (R 3/08). Similarly, the board had announced that it was not admitting the documents relatively early, yet the appellant had waited to raise its objection until the board had informed the parties of its negative opinion concerning sufficiency of disclosure. By adopting a wait-and-see attitude, the appellant constrained the board's possibilities to react to the objection raised subsequently under R. 106 EPC.

7.2.4 Contents of the petition for review in accordance with Rule 107 EPC

In R 4/12, the petitioner alleged that the board of appeal had failed to deal with the contention that D1 described means of accessing information but not services. Since this complaint had not been raised in the petition for review, it was entirely new and could therefore have no bearing whatsoever on the findings as to the petition's admissibility and merits reached on the basis of the grounds, facts, arguments and evidence set out in the petition.

7.2.5 Fundamental violation of Article 113 EPC (Article 112a(2)(c) EPC)

a) Situations involving no fundamental violation of Article 113 EPC

(i) Role of the boards of appeal

The Enlarged Board of Appeal rejected the petitioner's premise in R 2/13; namely that in ex parte cases the board is under an obligation to make the appellant aware of the board's reasons and arguments and thereby perform the role usually incumbent upon the opposing party in inter partes cases. This had no legal basis; it was a well-established principle that one cannot be judge and party. Whilst it was true that the parties have a right to comment upon the points which will be decisive for the decision, the view that this right also extends to all the sequences of reasoning (the "arguments" within the meaning of G 4/92, OJ EPO 1994, 149) was based on a misunderstanding of the terminology used. To require that the board should have made the petitioner aware of the particular steps in its reasoning went too far, as had been reiterated many times in the case law (see e.g. R 18/09 and R 10/10). Nor is the deciding body under an obligation to address each and every argument presented by the party (R 19/10, R 13/12, R 6/11); if one ground was found to render the invention obvious, the lines of reasoning supporting inventive step did not need to be addressed.

In R 12/12, the Enlarged Board pointed out that, where a party learned of possible new facts which had not or only partly been disclosed, it had to ask the board for an explanation. To ignore the opportunity to do so and then file a petition for review was tantamount to the conduct regarded as potentially unfair by the Enlarged Board in R 21/11. It was incumbent on the party seeking a decision in its favour to participate actively in proceedings and to do so at the appropriate time and on its own initiative (following R 2/08).

(ii) Substantive reasons

In R 4/12, the Enlarged Board found that remarks made by the board chairman shortly before delivering the decision, which are often simply for information purposes, are not grounds for the decision. They are not made to that end, are generally not recorded in the official minutes and do not usually result from any thorough analysis. What matters are the written reasons for dismissing the appeal. It is also important to ascertain whether the appellant had an opportunity to comment on those reasons and/or whether the reasons take due account of the arguments invoked by the appellant during the proceedings.

(iii) Presenting comments

In R 13/12 the petitioner made it quite clear that the core of the petition was not that it had not had sufficient opportunity to present its case, but that the decision did not consider and discuss the key arguments it had presented. The Enlarged Board agreed that it is undisputed that the right to be heard is not a purely formal right to speak, but implies an obligation on the boards of appeal to consider the parties' argumentation. This obligation is shaped by the circumstances of each case. This means that the boards have an obligation to discuss in their decisions issues and arguments to the extent that they are relevant for the decision and may disregard irrelevant arguments. The boards are not obliged to use specific words or the same wording as the parties and it must be accepted that refutation of arguments may be implicitly inferred from the particular reasoning held by the boards. In the case in suit, the Enlarged Board, although having no jurisdiction to review the substantive assessments, was satisfied that the decision under review had considered the petitioner's arguments.

Investigating further would have involved assessing whether the board had correctly understood the argumentation and given the correct answer, which was a matter for a review of the merits of the decision (a means of redress which does not exist in the EPC). Such a request therefore fell outside the ambit of the petition for review as intended by the legislator (see e.g. R 4/11). The Enlarged Board had to remain vigilant and thwart any attempt to blur the frontier between what might clearly be a matter of a violation of the right to be heard under Art. 113 and 112a(2)(c) EPC (for instance, an evident failure to consider an important factual submission) and anything else presented as a violation of the right to be heard but which actually pertained to the substantive merits of the decision.

In R 1/12 the Enlarged Board of Appeal made clear that observance of the right to be heard did not necessarily require that the party be heard on all questions of fact and law it considered important. Rather, the right to comment within the meaning of Art. 113(1) EPC related to the reasons for the decision, which were a matter for the decision-making body. Any error it made in those reasons was a question of the merits of the decision and not a breach of Art. 113(1) EPC, just as the restriction or even denial of the right to comment on facts and legal aspects irrelevant to the (fundamental) reasons for the decision could not constitute such a breach. On the contrary, in the interests of a fair, efficient and proper conduct of the proceedings, the chairperson of the decision-making body was practically obliged to intervene to prevent the submission, by any of the parties, of comments which were rambling, repetitive or irrelevant (or no longer relevant).

(iv) Party's requests

The Enlarged Board in R 17/11 held that it is not for the boards to ensure, of their own motion, that all points raised at some point in the proceedings are discussed at the oral proceedings.

Rather, it is for the parties to address any point they consider relevant and fear may be overlooked and to insist, if necessary by way of a formal request, that it be discussed. If a board then denies the party an opportunity to put forward its arguments, this may constitute a ground for asserting a breach of the right to be heard under Art. 113(1) EPC. This was confirmed in R 9/11 and R 11/12.

Following R 17/11, the Enlarged Board of Appeal also confirmed in R 9/11 that a board's decision on whether or not to admit a late-filed request is primarily discretionary and so can only be reviewed to the extent that the way in which a board exercised its discretion constituted a fundamental violation of Art. 113 EPC. Whether the decision is right or not and whether the Enlarged Board would have reached the same decision are not the relevant criteria.

In R 1/13 the petitioner argued that the right to be heard under Art. 113 EPC enshrines the possibility of a full reaction to overcome any late-raised objection, a right which can only be safeguarded by allowing a full discussion on the merits of new requests filed in response to a late objection; which in turn can only be achieved by admitting such requests. The Enlarged Board of Appeal disagreed. Art. 113 EPC simply provides that decisions may not be based on matters on which parties have not had an opportunity to comment – in this case whether or not to admit the petitioner's new requests, and on that point the petitioner had had the opportunity to comment.

(v) Comments by examiner during oral proceedings before the board of appeal

The petitioner in R 4/12 asserted that it had been surprised to learn, during the break in the oral proceedings, from the only third party attending them, who had presented himself as the examiner who had taken the decision to refuse the application, that its appeal was going to be dismissed, before the registrar had even announced resumption of the proceedings. The Enlarged Board, however, considered that this assertion had no bearing on the petitioner's submissions with regard to infringement of its right to be heard under Art. 113(1) EPC.

(vi) Causal link

In R 21/11, the petitioner had argued that the test applicable in deciding whether its allegedly non-decided request had been relevant within the meaning of R. 104(b) EPC was not whether the outcome would actually have been different if it had been decided on but, rather, whether it could potentially have led to a different outcome. The Enlarged Board largely agreed: if a non-decided request could not have affected the outcome of the appeal, it would be pointless to allow a petition for review, even if it were clearly established that the request had not been addressed. If, on the other hand, a request were only to be considered relevant where it could be ascertained beyond any doubt that the outcome would have been different, R. 104(b) EPC would become largely redundant (quite apart from the need to re-examine the facts). In the Enlarged Board's view, a request "relevant" to the decision within the meaning of R. 104(b) EPC had to be defined as one capable of altering the outcome.

This was in keeping with the Enlarged Board's settled case law on infringement of the right to be heard, according to which one condition was that there be a causal link between the procedural defect constituting the infringement and the decision in that the defect was decisive factor in reaching that decision (settled case law following R 1/08), i.e. it could not be ruled out that a different decision would have been reached if the party had been heard on the point with respect to which it alleged that its right to be heard had been infringed. This condition was met in the case now before the Enlarged Board.

The Enlarged Board pointed out in R 4/12 that Art. 113(1) EPC could not be regarded as infringed simply because the decision dealt with arguments which, although liable to work in the appellant's favour, had possibly never been part of the grounds invoked. In other words, there was no causal link between the alleged procedural defect and the decision.

b) Situations involving a fundamental violation of Article 113 EPC

In the case underlying R 15/11 the board of appeal had found that the auxiliary request did not comply with the requirements of Art. 84 EPC. However, there was no explicit or implicit indication in the file from which it could be derived that a possible lack of clarity had ever been discussed or any objection raised. The Enlarged Board said it did not have the power to investigate further whether the petitioner was or could have been aware of the board's doubts. In the absence of any such indication, it was clearly not for the party which alleged a breach of its right to be heard under Art. 113 EPC to prove the negative (negativa non sunt probanda).

It was for the board to draft its own texts in a way that enabled the reader, taking into account all the documents on file, to conclude that the right to be heard within the meaning of Art. 113(1) EPC had been respected with regard to the grounds on which the decision of the board was based. The Enlarged Board was not in a position to establish that the petitioner's right to be heard had been respected. It had therefore to be assumed that a violation of the petitioner's rights under Art. 113(1) EPC had occurred which qualified as fundamental within the meaning of Art. 112a(2)(c) EPC because it concerned the ground on which the appeal was eventually dismissed by the decision under review. The petition was therefore allowed.

See also R 21/11 below.

7.2.6 Any other fundamental procedural defect (Article 112a(2)(d) EPC)

(a) Party's requests under R. 104(b) EPC

(i) Decision on request under Rule 104(b) EPC

In R 21/11 the petitioner claimed that:

(1) The board had decided on the appeal without deciding a request relevant to the decision, namely that for admission of a second expert testimony filed by fax, (Art. 112a(2)(d) EPC and R. 104(b) EPC) and, moreover, its failure to take account of this testimony in its decision had infringed the patentee's right to be heard (Art. 112a(2)(c) EPC and Art. 113(1) EPC).

(2) Its right to be heard had also been infringed in so far as it had not been given sufficient opportunity (only five days) to comment on the objection, first raised in a communication from the board, of a lack of novelty over D8. This second ground was found inadmissible (see point 7.2.3 b) above).

The Enlarged Board found that, when interpreting R. 104(b) EPC, it had to be borne in mind that the provision made for asserting a failure to decide a request as a ground for review was a specific manifestation of the right to be heard, which right applied regardless of whether or not the party's submission might actually have persuaded the board. Generally speaking, it would be wrong to define relevance for the purposes of R. 104(b) EPC so narrowly that the ground set out there could not be established where that under Art. 112a(2)(c) EPC could be established on the same facts. Such an argument that the undecided request was irrelevant because its admission would have made no difference could be accepted only if it could be shown that all the aspects "lost" as a result of its non-admission had been taken into account in the decision under review.

In the Enlarged Board's view, "relevant" for the purposes of R. 104(b) EPC had to be defined as capable of changing the outcome of the appeal.

For an infringement of the right to be heard to be established, there had to be a causal link between the procedural defect alleged to constitute that infringement and the decision taken in that the defect was decisive in reaching the decision. This requirement was met in the case at hand (see point 7.2.5 a) above).

According to the Enlarged Board, the first procedural defect asserted by the petitioner constituted an infringement of its right to be heard (Art. 112a(2)(c) EPC in conjunction with Art. 113(1) EPC). While the petitioner had argued that this infringement consisted in the board's failure to take account of the second testimony in its decision, the Enlarged Board saw the infringement in the board's failure to consider the request for its admission. Had that request been granted, it could have influenced the outcome, so that there was a causal link between the denied opportunity to comment and the board's decision. The contested decision was therefore set aside.

7.3 Examination of the petition under Rule 108 EPC

7.3.1 Replacement of members of the board of appeal under Rule 108(3) EPC

Where a petition for review is allowable, "the Enlarged Board of Appeal may order that members of the Board of Appeal who participated in taking the decision set aside shall be replaced" (R. 108(3) EPC). The Enlarged Board ruling in R 21/11 considered the use of "may" ("peut" in French; "kann" in German) significant because it left to its discretion whether or not to replace board members. It had to exercise this discretion fairly and proportionately in the light of the facts. If, for instance, it transpired that the members who had participated in taking the decision reviewed were, or might have been, biased it might be appropriate to order their replacement. But in the absence of any acceptable reason to do so, no such order was to be made, not least to prevent any increase in the costs and duration of proceedings as a result of their reopening. In the case at hand, the Enlarged Board could find no indication of bias and so the board members were not replaced.

In R 15/11 the petitioner argued that the replacement of all members of the board of appeal should be an automatic measure whenever the violation of the petitioner's rights which led to the setting aside of the decision of the board was a mistake of the board itself, and not, as in R 21/11, due to circumstances outside the board's control. The Enlarged Board disagreed. Such a narrow understanding of the Enlarged Board's discretion to replace members of the boards (or not) could not be derived from the wording of R. 108(3) EPC, nor did it properly reflect its purpose. On the contrary, the rule was that the proceedings were re-opened before the board responsible under the business distribution scheme.

The Enlarged Board went on to say that the principle relied on in R 21/11, namely that the replacement of members of the board of appeal should not be ordered without good reason, was not merely a matter of procedural economy. Adherence to the business distribution scheme was an important element of an independent, reliable and efficient judicial system meeting the requirements of, inter alia, Art. 6 ECHR.

It was accepted that the suspicion of partiality must be justified on an objective basis. The Enlarged Board found that such a suspicion was not objectively justified for the sole reason that a position on a matter had been adopted in a prior decision of a board in which the board member concerned had participated (following G 1/05, OJ EPO 2008, 271 and J 15/04).

297 references found. Limiting display to 200 references.

Click X to load a reference inside the current page, click on the title to open in a new page.

Case Law of the Enlarged Board

General Case Law